ORTHOPEDIC MCQS WITH ANSWER TUMOR/ONCOLOGY 05

ORTHOPEDIC MCQS WITH ANSWER TUMOR/ONCOLOGY 05

 

1.      A 10-year-old child has leg discomfort with activity.  A radiograph, bone scan, and biopsy specimen are shown in Figures 1a through 1c.  What is the most likely diagnosis?

 

1-         Parosteal osteosarcoma

2-         Unicameral bone cyst

3-         Aneurysmal bone cyst

4-         Eosinophilic granuloma

5-         Fibrous dysplasia

 

PREFERRED RESPONSE: 5

 

DISCUSSION: The ground glass appearance on the radiograph, the hot bone scan, and histologic findings of bony spicules without osteoblastic rimming in a background of bland fibrous tissue all suggest fibrous dysplasia.  Stress-related pain is common with activity because of the dysplastic bone.  Parosteal osteosarcomas are surface lesions.  Simple cysts, aneurysmal bone cysts, and eosinophilic granuloma are all possible radiographically; however, the histology is most consistent with fibrous dysplasia.

 

REFERENCES: Harris WH, Dudley HR Jr, Barry RS: The natural history of fibrous dysplasia: An orthopaedic, pathological and roentgenographic study.  J Bone Joint Surg Am 1962;44:207.

Campanacci M: Bone and Soft Tissue Tumors.  Vienna, Austria, Springer-Verlag, 1990.

 

 

2.        A 13-year-old boy has pain and a firm mass in his left knee.  A radiograph and MRI scan are shown in Figures 2a and 2b, and a biopsy specimen is shown in Figure 2c.  Based on these findings, what is the most likely diagnosis?

 

1-         Osteosarcoma

2-         Osteochondroma

3-         Ewing’s sarcoma

4-         Chondrosarcoma

5-         Periosteal chondroma

 

PREFERRED RESPONSE: 1

 

DISCUSSION: The most likely diagnosis is osteosarcoma.  The imaging studies show an aggressive primary tumor of bone, and the histology slide shows a typical chondroblastic osteosarcoma, with osteoid deposited along the surface of bone trabeculae.  Ewing’s sarcoma histologically consists of small round blue cells. Osteochondroma and periosteal chondroma can occur near the knee but have different radiographic and histologic patterns.  Chondrosarcoma rarely occurs in children.

 

REFERENCES: Simon M, Springfield D, et al: Osteogenic sarcoma: Surgery for Bone and Soft Tissue Tumors.  Philadelphia, PA, Lippincott Raven, 1998, p 267.

Wold LA, et al: Atlas of Orthopaedic Pathology.  Philadelphia, PA, WB Saunders, 1990,

pp 14-15.

 

3.        A 10-year-old boy with a history of retinoblastoma now reports right knee pain.  AP and lateral radiographs are shown in Figures 3a and 3b.  What is the most likely diagnosis?

 

1-         Ewing’s sarcoma

2-         Primitive neuroectodermal tumor

3-         Osteosarcoma

4-         Osteonecrosis

5-         Osteomyelitis

 

PREFERRED RESPONSE: 3

 

DISCUSSION: The radiographs show a bone-producing lesion in the femoral diaphysis. The radiographic appearance of small round cell tumors is more permeative with an elevated periosteum and no matrix production. The appearance of this lesion is most consistent with osteosarcoma.  Patients who carry the Rb gene are predisposed to osteosarcoma.  However, Ewing’s sarcoma, primitive neuroectodermal tumor, and osteomyelitis can all occur in this location.  

 

REFERENCES: Unni KK: Dahlin’s Bone Tumors: General Aspects and Data on 11,087 Cases, ed 5.  Philadelphia, PA, Lippincott-Raven, 1996, pp 143-160.

Chauveinc L, Mosseri V, Quintana E, Desjardins L, Schlienger P, Doz F, Dutrillaux B: Osteosarcoma following retinoblastoma: Age at onset and latency period.  Ophthalmic Genet 2001;22:77-88.

 

4.        Which of the following tumors have characteristic chromosomal translocations?

 

1-         Ewing’s sarcoma and osteosarcoma

2-         Ewing’s sarcoma and conventional chondrosarcoma

3-         Ewing’s sarcoma and synovial sarcoma

4-         Osteosarcoma and conventional chondrosarcoma

5-         Myeloma and malignant fibrous histiocytoma

 

PREFERRED RESPONSE: 3

 

DISCUSSION: Ewing’s sarcoma has an 11;22 translocation that creates the EWS/FLI1 fusion gene, and synovial sarcoma has an X;18 translocation that creates the STT/SSX fusion gene.  The other tumors do not have consistent translocations.

 

REFERENCES: Sandberg AA: Cytogenetics and molecular genetics of bone and soft-tissue tumors.  Am J Med Genet 2002;115:189-193.

Menendez LR (ed): Orthopaedic Knowledge Update: Musculoskeletal Tumors.  Rosemont, IL, American Academy of Orthopaedic Surgeons, 2002, pp 11-20.

 

5.        What is the most common MRI appearance of a malignant soft-tissue sarcoma?

 

1-         Well defined, homogeneous, and deep to fascia

2-         Well defined, heterogeneous, and deep to fascia

3-         Ill defined, homogeneous, and deep to fascia

4-         Ill defined, heterogeneous, and superficial to fascia

5-         Ill defined, homogeneous, and superficial to fascia

 

PREFERRED RESPONSE: 2

 

DISCUSSION: The classic MRI appearance of a soft-tissue sarcoma is a well-defined heterogeneous mass deep to the fascia.  MRI has greatly enhanced our ability to identify and characterize soft-tissue masses.  In many patients, MRI is diagnostic and may obviate the need for biopsy.  In other patients, it may indicate with high probability that the mass is malignant and consideration for referral can be made.  A common misconception is that sarcomas are infiltrative; therefore, physicians mistakenly exclude the diagnosis of a sarcoma based on a well-defined mass seen on MRI.  However, sarcomas grow centrifugally with balloon-like expansion compressing surrounding normal tissue; as such, they appear well defined.  Many benign soft-tissue masses such as lipomas are similarly well defined.  However, MRI is especially useful in identifying fat.  Lipomas appear to be homogeneous masses with fat signal characteristics on all sequences.  Ill-defined soft-tissue masses include infection, trauma, and desmoid tumors.  Heterogeneity is not unique to malignant tumors but is a characteristic of soft-tissue sarcomas.

 

REFERENCES: Bancroft LW, Peterson JJ, Kransdorf MJ, Nomikos GC, Murphey MD: Soft tissue tumors of the lower extremities.  Radiol Clin North Am 2002;40:991-1011.

Berquist TH, Ehman RL, King BF, et al: Value of MR imaging in differentiating benign from malignant soft-tissue masses: Study of 95 lesions.  Am J Roentgenol 1990;155:1251-1255.

Crim JR, Seegar LL, Yao L, et al: Diagnosis of soft tissue masses with MR imaging: Can benign masses be differentiated from malignant ones?  Radiology 1992;185:581-586.

 

6.        Which of the following factors is associated with the worst prognosis in soft-tissue sarcomas?

 

1-         Size greater than 15 cm

2-         Extra-compartmental involvement

3-         Number of mitotic figures per high-power field (grade)

4-         Large size in a proximal location

5-         Presence of metastases

 

PREFERRED RESPONSE: 5

 

DISCUSSION: Although factors such as a high-grade tumor and large size are associated with decreased survival, the presence of metastases carries the worst prognosis.  Good results are very rare when metastases are present.  Soft-tissue sarcomas, as a whole, respond poorly to chemotherapy, leading to a poor prognosis when metastases are present.

 

REFERENCES: Collin C, Goobold J, Hadju SI, Brennan MF: Localized extremity soft tissue sarcoma: An analysis of factors affecting survival.  J Clin Oncol 1987;5:601-612.

Eilber FC, Rosen G, Nelson SE, et al: High-grade extremity soft tissue sarcomas: Factors predictive of local recurrence and its effect on morbidity and mortality. 

Ann Surg 2003;237:218-226.

 

7.        The lesion seen in Figure 4 is most likely the result of metastases from what solid organ?

 

1-         Breast

2-         Lung

3-         Thyroid

4-         Prostate

5-         Liver

 

PREFERRED RESPONSE: 2

 

DISCUSSION: The primary carcinoma most likely to metastasize distal to the elbow and knees is lung carcinoma.  Renal cell carcinoma can also metastasize to distal sites.  Most metastatic bone disease occurs in the vertebral bodies, pelvis, and proximal long bones. 

 

REFERENCES: Simon MA, Bartucci EJ: The search for the primary tumor in patients with skeletal metastases of unknown origin.  Cancer 1986;58:1088-1095.

Leeson MC, Makley JT, Carter JR: Metastatic skeletal disease distal to the elbow and knee.  Clin Orthop 1986;206:94-99.

 

8.        A 7-year-old boy has a limp with pain and tenderness over the distal right femur.  Radiographs are shown in Figures 5a and 5b.  Based on these findings, what is the best course of action?

 

1-         Observation

2-         Radiofrequency ablation

3-         Biopsy, curettage, and bone grafting

4-         Radiation therapy

5-         Steroid injection

 

PREFERRED RESPONSE: 3

 

DISCUSSION: The patient has a nonossifying fibroma, and the symptoms suggest that there is significant weakening of the cortex and/or microfracture; therefore, biopsy, curettage, and bone grafting is the treatment of choice.  Based on the size of the lesion and the presence of symptoms, observation is likely to result in pathologic fracture.  A biopsy will further weaken the bone unless combined with curettage and bone grafting.  Radiation therapy is not used in the treatment of nonossifying fibroma and is rarely used in any benign condition affecting skeletally immature individuals.  Steroid injection has been used in the treatment of unicameral bone cysts and in Langerhan’s cell histiocytosis but not for nonossifying fibroma.

 

REFERENCES: Dormans, JP, Pill SG: Fractures through bone cysts: Unicameral bone cysts, aneurysmal bone cysts, fibrous cortical defects, and nonossifying fibromas.  Instr Course Lect 2002;51:457-467.

Easley ME, Kneisl JS: Pathologic fractures through nonossifying fibromas: Is prophylactic treatment warranted?  J Pediatr Orthop 1997;17:808-813.

 

9.        A 14-year-old patient has anterior knee pain.  Radiographs, an MRI scan, and biopsy specimens are shown in Figures 6a through 6e.  What is the most likely diagnosis?

 

1-         Unicameral bone cyst

2-         Aneurysmal bone cyst

3-         Giant cell tumor

4-         Chondromalacia

5-         Ewing’s sarcoma

 

PREFERRED RESPONSE: 5

 

DISCUSSION: Although the imaging studies are consistent with a unicameral bone cyst, aneurysmal bone cyst, or giant cell tumor, the histology shows small round blue cells that are typical of Ewing’s sarcoma.  Although Ewing’s sarcoma frequently occurs in the diaphysis, it can occur in the metaphysis.

 

REFERENCE: Mirra J: Bone Tumors: Clinical, Radiologic, and Pathologic Correlations.  Philadelphia, PA, Lea & Febiger, 1989, vol 2, ch 18.

 

10.      A 47-year-old woman has an asymptomatic pelvic mass that was discovered on routine gynecologic examination.  A radiograph, CT scan, MRI scan, and biopsy specimen are shown in Figures 7a through 7d.  Metastatic work-up is negative.  Treatment should consist of

 

1-         observation.

2-         primary wide resection.

3-         intralesional curettage.

4-         radiation therapy.

5-         preoperative chemotherapy.

 

PREFERRED RESPONSE: 2

 

DISCUSSION: The imaging studies show a chondrosarcoma; therefore, surgical treatment is indicated.  There is no role for intralesional treatment of an exophytic lesion, particularly in the pelvis.  Even obtaining a biopsy specimen risks intrapelvic contamination, although many surgeons would still perform a biopsy prior to a resection to confirm the diagnosis.  Chondrosarcoma is considered resistant to both radiation therapy and chemotherapy; therefore, radiation therapy generally is not used except for unresectable lesions.  Chemotherapy would be used only for metastatic disease or in patients with high-grade chondrosarcoma.  The grade would not be known until after resection, and in this patient, the histology slide showed a grade I neoplasm.  Chemotherapy would not be used preoperatively because a cartilage tumor is unlikely to shrink, and in this patient, the lesion is resectable.

 

REFERENCES: Springfield DS, Gebhardt MS, Mcguire MH: Chondrosarcoma: A review.  J Bone Joint Surg Am 1996;78:141-149.

Marco RA, Gitelis S, Brebach GT, Healey JH: Cartilage tumors: Evaluation and treatment.  J Am Acad Orthop Surg 2000;8:292-304.

 

11.       A 20-year-old patient has foot pain.  A radiograph and T1-weighted MRI scan are shown in Figures 8a and 8b.  A biopsy specimen is shown in Figure 8c.  Treatment should consist of

 

1-         extended curettage and cementation.

2-         amputation of the first ray.

3-         wide resection and chemotherapy.

4-         extended curettage, radiation therapy, and chemotherapy.

5-         Syme’s amputation.

 

PREFERRED RESPONSE: 1

 

DISCUSSION: Giant cell tumors occur near articular surfaces in young adults.  The histology shows abundant giant cells with nuclei resembling the surrounding cells.  Although the MRI scan shows soft-tissue involvement, curettage is still the preferred treatment.  Chemotherapy is not necessary for benign lesions, and amputation is too aggressive.  Cementation, phenol, and cryosurgery (liquid nitrogen) are all acceptable local adjuvants to curettage.  Packing the cavity with bone graft rather than cement is also acceptable.

 

REFERENCES: Dahlin DC, Unni KK: Bone Tumors: General Aspects and Data on 8,542 Cases.  Springfield, IL, Charles C. Thomas, 1986.

Gitelis S, Mallin BA, Piasecki P, Turner F: Intralesional excision compared with en bloc resection for giant cell tumor of bone.  J Bone Joint Surg Am 1993;75:1648-1655.

 

12.      A 69-year-old man has a painful slow-growing lesion of the distal phalanx of his thumb.  History reveals that he has had chronic osteomyelitis of the thumb for the past 12 years.  The radiograph and biopsy specimens are seen in Figures 9a through 9c.  Treatment should consist of

 

1-         intralesional curettage.

2-         wrist disarticulation.

3-         amputation.

4-         chemotherapy.

5-         radiation therapy.

 

PREFERRED RESPONSE: 3

 

DISCUSSION: The diagnosis is squamous cell carcinoma.  The radiograph shows a destructive lesion, and the histologic slides demonstrate squamous cells invading bone.  The preferred treatment for squamous cell carcinoma is wide resection; however, in this location a wide margin can be achieved only with amputation.  Overall survival in patients with squamous cell carcinoma secondary to chronic osteomyelitis is not significantly worse than that expected for age-matched controls.

 

REFERENCES: Dell PC: Hand, in Simon MA, Springfield D (eds): Surgery for Bone and Soft Tissue Tumors.  Philadelphia, PA, Lippincott-Raven, 1998, pp 405-420.

McGrory JE, Pritchard DJ, Unni KK, Ilstrup D, Rowland CM: Malignant lesion arising in chronic osteomyelitis.  Clin Orthop 1998;362:181-189.

 

13.       A 12-year-old girl has had right knee pain for the past 3 months.  Radiographs and a coronal T2-weighted MRI scan are shown in Figures 10a through 10c.  A biopsy specimen is shown in Figure 10d.  What is the most appropriate treatment for this lesion?

 

1-         Chemotherapy and radiation therapy

2-         Neoadjuvant chemotherapy followed by wide resection and reconstruction

3-         Curettage and bone grafting

4-         Curettage and intravenous antibiotics

5-         Above-knee amputation

 

PREFERRED RESPONSE: 2

 

DISCUSSION: The radiographs show a lytic lesion in the right proximal tibia that has a high fluid content based on the MRI findings.  The radiographic appearance is consistent with either telangiectatic osteosarcoma or aneurysmal bone cyst.  Low-magnification histology shows a lesion resembling an aneurysmal bone cyst, with blood lakes separated by cellular septa.  However, high-magnification shows severe cytologic atypia, indicative of a telangiectatic osterosarcoma.  The appropriate treatment is neoadjuvant chemotherapy followed by wide resection and reconstruction of the lesion. 

 

REFERENCES: Unni KK: Dahlin’s Bone Tumors: General Aspects and Data on 11,087 Cases, ed 5.  Philadelphia, PA, Lippincott-Raven, 1996, pp 143-160.

Arndt CA, Crist WM: Common musculoskeletal tumors of childhood and adolescence.  N Engl J Med 1999;341:342-352.

 

14.      Which of the following diagnostic studies best distinguishes Ewing’s sarcoma from small cell osteosarcoma?

 

1-         Bone scan

2-         Cytogenetics

3-         DNA ploidy analysis

4-         Sentinel node biopsy

5-         L-lactate dehydrogenase (LDH)

 

PREFERRED RESPONSE: 2

 

DISCUSSION: Cytogenetics best demonstrates the 11;22 translocation characteristic of Ewing’s sarcoma.  The translocation also can be detected with polymerase chain reaction and fluorescent in situ hybridization.  The Ewing antibody is used for immunostaining to check for cell membrane (surface) staining of a marker unrelated to the translocation; this could also help distinguish Ewing’s sarcoma from small cell osteosarcoma.  A bone scan will show increased uptake with both types of tumors.  Although most Ewing’s sarcoma tumors are diploid, some are polyploid as are most osteosarcomas.  Flow cytometry is used to sort cells, sometimes based on antibody binding.  LDH can be elevated in both Ewing’s sarcoma and osteosarcoma and is a poor prognostic indicator when elevated.  Lymph node metastases are uncommon in both of these tumors. 

 

REFERENCES: Perotti D, Corletto V, Giardini R, Parafioriti A, Fossati-Bellani F, Luksch R: Retrospective analysis of ploidy in primary osseous and extraosseous Ewing family tumors in children.  Tumori 1998;84:493-498.

Riley RD, Burchill SA, Abrams KR, Heney D, Sutton AJ, Jones DR, et al: A systematic review of molecular and biological markers in tumours of the Ewing’s sarcoma family.  Eur J Cancer 2003;39:19-30.

Menendez LR (ed): Orthopaedic Knowledge Update: Musculoskeletal Tumors.  Rosemont, IL, American Academy of Orthopaedic Surgeons, 2002, pp 11-20.

 

15.      Figure 11a shows the AP pelvis radiograph of a 25-year-old man who sustained a spinal cord injury 10 years ago.  A bone scan and a CT scan are shown in Figures 11b and 11c.  To prevent recurrence after resection, management should consist of

 

1-         radiation therapy.

2-         bisphosphonate therapy.

3-         antitubercular therapy.

4-         parathyroidectomy.

5-         a diet low in phosphate.

 

PREFERRED RESPONSE: 1

 

DISCUSSION: The studies reveal significant heterotopic ossification that appears to be mature.  Following resection, the most reliable way to prevent recurrence is with low-dose external-beam radiation therapy.  Bisphosphonate therapy can be considered; however, when terminated, heterotopic bone may reform.  Heterotopic ossification is unrelated to the patient’s endocrine status and is not associated with any metabolic abnormalities. 

 

REFERENCES: Moore K, Goss K, Anglen J: Indomethacin versus radiation therapy for prophylaxis against heterotopic ossification in acetabular fracture.  J Bone Joint Surg Br 1998;80:259.

Stover S, Niemann K, Tullos J: Experience with surgical resection of heterotopic bone in spinal cord injury patients.  Clin Orthop 1991;263:71-77.

 

 

 

16.       A 40-year-old man has a palpable mass over the dorsum of the ankle.  He reports no history of direct trauma but notes that he sustained a laceration to the middle of his leg 6 weeks ago.  Examination reveals a 4-cm x 1-cm mass.  T1- and T2-weighted MRI scans are shown in Figures 12a and 12b.  An intraoperative photograph and biopsy specimen are shown in Figures 12c and 12d.  What is the most likely diagnosis?

 

1-         Giant cell tumor of the tendon sheath

2-         Gouty tophi

3-         Aggressive fibromatosis

4-         Tendon rupture

5-         Synovial cell sarcoma

 

PREFERRED RESPONSE: 4

 

DISCUSSION: The findings are most consistent with a rupture of the anterior tibial tendon.  The damaged area of tendon should be resected, followed by tendon reconstruction or tenodesis.  The histology is not consistent with giant cell tumor of the tendon sheath, gout, or synovial sarcoma.  Fibromatosis is characterized by a large number of spindle cells within the collagen background.

 

REFERENCES: Otte S, Klinger HM, Loreaz F, Haerer T: Operative treatment in case of closed rupture of the anterior tibial tendon.  Arch Orthop Traum Surg 2002;122:188-190.

Kausch T, Rutt J: Subcutaneous rupture of the tibialis anterior tendon: Review of the literature and case report.  Arch Orthop Traum Surg 1998;117:290-293.

 

17.      What is the most common presentation of a benign bone tumor in childhood?

 

1-         Pain

2-         Deformity

3-         Pathologic fracture

4-         Presence of a mass

5-         Incidental finding

 

PREFERRED RESPONSE: 5

 

DISCUSSION: The most common benign bone tumors in childhood are discovered incidentally and include single bone cysts, fibrous cortical defects, nonossifying fibroma, and osteochondroma.  Benign bone tumors can be classified as latent, active, or aggressive.  Aggressive bone tumors usually present with pain, whereas active lesions present with pain or pathologic fracture.  Only aggressive benign bone tumors are associated with a soft-tissue mass, and they are far less common than indolent bone tumors, especially in children.   

 

REFERENCES: Aboulafia AJ, Kennon RE, Jelinek JS: Benign bone tumors of childhood.  J Am Acad Orthop Surg 1999;7:377-388.

Biermann JS: Common benign lesions of bone in children and adolescents.  J Pediatr Orthop 2002;22:268-273.

 

18.      Figures 13a and 13b show the MRI scans of a 70-year-old patient who has a posterior calf mass.  Examination reveals that the mass extends to the midcalf level.  A biopsy specimen reveals a high-grade soft-tissue sarcoma.  Metastatic work-up shows no lesions.  Management should consist of

 

1-         wide resection and radiation therapy.

2-         wide resection and chemotherapy.

3-         below-knee amputation.

4-         observation.

5-         radiation therapy.

 

PREFERRED RESPONSE: 3

 

DISCUSSION: Soft-tissue sarcomas generally are treated with radiation therapy and wide surgical resection.  In this patient, involvement of most of the posterior calf compartment and circumferential involvement of the posterior tibial and peroneal neurovascular bundle makes limb salvage impractical.  Any attempt at wide surgical resection would leave a poorly functioning limb with questionable surgical margins.  A high below-knee amputation would be the best option.  Radiation therapy alone is contraindicated.

 

REFERENCES: Lindberg RD, Martin RG, Romsdahl MM, et al: Conservative surgery and post-operative radiotherapy in 300 adults with soft tissue sarcoma.  Cancer 1981;47:2391-2397.

Sim FT, Frassica FS, Frassica DA: Soft tissue tumors: Diagnosis, evaluation, and management.  J Am Acad Orthop Surg 1994;2:202-211.

Rosenberg SA, Tepper J, Glatstein E, et al: The treatment of soft-tissue sarcomas of the extremities: Prospective randomized evaluations of (1) limb sparing surgery plus radiation therapy compared with amputation and (2) The roll of adjuvant chemotherapy.  Am Surg 1982;196:305-315.

 

19.      What is the most common secondary malignancy arising in pagetic bone?

 

1-         Fibrosarcoma

2-         Chondrosarcoma

3-         Malignant fibrous histiocytoma

4-         Osteosarcoma

5-         Ewing’s sarcoma

 

PREFERRED RESPONSE: 4

 

DISCUSSION: The incidence of malignant transformation or the formation of a secondary malignancy in pagetic bone is estimated to be less than 1%.  Osteosarcoma is the most common secondary malignancy, followed by fibrosarcoma and chondrosarcoma.  Ewing’s sarcoma arising in pagetic bone has not been reported.  

 

REFERENCES: Grimer RJ, Carter SR, Tillman RM, et al: Osteosarcoma of the pelvis.  J Bone Joint Surg Br 1999;81:796-802.

Harrington KD: Surgical management of neoplastic complications of Paget’s disease.  J Bone Miner Res 1999;14:45-48.

 

20.     A 15-year-old girl with a midshaft fibular lesion has histologic findings consistent with Ewing’s sarcoma.  Following induction chemotherapy, local control typically consists of

 

1-         radiation therapy only.

2-         curettage and bone grafting.

3-         wide resection.

4-         below-knee amputation.

5-         observation.

 

PREFERRED RESPONSE: 3

 

DISCUSSION: The current treatment regimen for Ewing’s sarcoma typically involves induction chemotherapy followed by local control and further chemotherapy.  Local control consists of surgery only, radiation therapy only, or a combination of the two.  In bones that are easily resectable (expendable) with wide margins, surgery is usually recommended.  For areas that cannot be resected (ie, large bulky pelvic tumors), radiation therapy alone is sometimes the preferred method of local control.  If surgery is chosen and the margins are close, radiation therapy can be used as an adjuvant treatment.  Amputation is rarely required for an isolated fibular lesion.  Observation without adequate local control results in local recurrence.

 

REFERENCES: Nesbit ME Jr, Gehan EA, Burgert EO Jr, et al: Multimodality therapy for the treatment of primary, non-metastatic Ewing’s sarcoma of the bone: A long-term follow-up of the first intergroup study.  J Clin Oncol 1990;8:1664-1674.

Simon M, Springfield D, et al: Ewing’s Sarcoma: Surgery for Bone and Soft Tissue Tumors.  Philadelphia, PA, Lippincott Raven, 1998, p 296.

 

 

21.      Soft-tissue sarcomas most commonly metastasize to the

 

1-         liver.

2-         lung.

3-         bone.

4-         regional nodes.

5-         distant nodes.

 

PREFERRED RESPONSE: 2

 

DISCUSSION: The most common location for soft-tissue tumors to metastasize is the lungs.  Depending on the grade of the sarcoma, metastases develop in as many as 50% of patients with soft-tissue sarcomas.  Tumor grade is considered the most significant prognostic factor in predicting risk of metastases, with high-grade lesions at greatest risk.  Staging CT of the chest should be performed once the diagnosis of a soft-tissue sarcoma is suspected or confirmed.  Regular surveillance of patients treated for soft-tissue sarcomas includes follow-up CT scans at regular intervals.  Intra-abdominal metastases are uncommon but may occur, particularly in patients with myxoid liposarcoma.  Regional metastases are relatively uncommon and occur in approximately 5% of all patients with soft-tissue sarcoma.  The incidence of regional nodal metastases is higher for synovial sarcoma and epithelioid sarcomas than other soft-tissue sarcomas.

 

REFERENCES: Sim FH, Frassica FJ, Frassica DA: Soft-tissue tumors: Diagnosis, evaluation, and management.  J Am Acad Orthop Surg 1994;2:202-211.

Enzinger FM, Weiss SW, Goldblum F: Soft Tissue Tumors, ed 4.  Washington, DC, Mosby/AFIP, 2001.

 

22.     The incidence of osteosarcoma is highest in what age group?

 

1-         First decade

2-         Second decade

3-         Fourth decade

4-         Fifth decade

5-         Sixth decade

 

PREFERRED RESPONSE: 2

 

DISCUSSION: The peak incidence of osteosarcoma occurs in the second decade, followed by the third decade.  Up to 75% of all cases of osteosarcoma occur in patients between 10 and 25 years.  It rarely occurs after age 30.  Affected women tend to be younger than affect men.  Osteosarcoma associated with Paget’s disease or in radiation-induced osteosarcoma occurs in an older population.

 

REFERENCES: Simon M, Springfield D, et al: Osteogenic Sarcoma: Surgery for Bone and Soft Tissue Tumors.  Philadelphia, PA, Lippincott Raven, 1998, p 266.

Mirra JM: Bone Tumors: Clinical, Radiologic, and Pathologic Correlations.  Philadelphia, PA, Lea and Febiger, 1989.

Wold L, et al: Osteogenic Sarcoma: Atlas of Orthopaedic Pathology.  Philadelphia, PA, WB Saunders, 1990, p 14.

 

23.     A 9-year-old boy has a painless enlarged mass on the dorsum of his hand.  Figures 14a through 14d show the clinical photograph, radiographs, and biopsy specimen.  What is the most likely diagnosis?

 

1-         Fracture callus

2-         Enchondroma (Ollier’s disease)

3-         Multiple hereditary exostosis

4-         Osteosarcoma

5-         Chondrosarcoma

 

PREFERRED RESPONSE: 3

 

DISCUSSION: Multiple hereditary exostosis and enchondroma commonly present as multiple lesions in the hand.  Multiple hereditary exostosis consists of cartilage capped bony exostoses arising from the metaphyseal end of rapidly growing bones.  Osteosarcoma and chondrosarcoma rarely appear as multiple lesions.  Fracture callus can exhibit enchondral ossification that is usually circumferential, but the radiographic findings are not consistent with fracture.

 

REFERENCES: Porter DE, Emerton ME, Villanueva-Lopez F, Simpson AH: Clinical and radiographic analysis of osteochondromas and growth disturbance in hereditary multiple exostoses.  J Pediatr Orthop 2000;20:246-250.

Pierz KA, Stieber JR, Kusumi K, Dormans JP: Hereditary multiple exostoses: One center’s experience and review of etiology.  Clin Orthop 2002;402:49-59.

 

24.     What is the most common clinical presentation of a patient with a malignant bone tumor?

 

1-         Incidental finding

2-         Pain

3-         Pathologic fracture

4-         Deformity

5-         Presence of a mass

 

PREFERRED RESPONSE: 2

 

DISCUSSION: The most common clinical presentation of a patient with a malignant bone tumor is pain.  Malignant bone tumors rarely are diagnosed as an incidental finding or pathologic fracture.  In patients who have a pathologic fracture on initial presentation, a history of increasing pain prior to the fracture is typical.  While 90% of malignant bone tumors are associated with a soft-tissue mass, in many patients the soft-tissue component of the tumor is not clinically apparent.

 

REFERENCES: Buckwalter JA: Musculoskeletal neoplasms and disorders that resemble neoplasms, in Weinstein SL, Buckwalter JA (eds): Turek’s Orthopaedics: Principles and Their Application, ed 5.  Philadelphia, PA, JB Lippincott, 1994, pp 290-295.

Mehlman CT, Crawford AH, McMath JA: Pediatric vertebral and spinal cord tumors: A retrospective study of musculoskeletal aspects of presentation, treatment, and complications. Orthopedics 1999;22:49-55.

 

25.     Figure 15a shows the radiograph of a patient who has a chondrosarcoma of the acetabulum.  Bone scans are shown in Figures 15b and 15c.  Numerous soft subcutaneous masses are present.  A clinical photograph of the hand is shown in Figure 15d.  What is the most likely diagnosis?

 

1-         Multiple hereditary exostosis

2-         Ollier’s disease

3-         Maffucci’s syndrome

4-         McCune-Albright syndrome

5-         Neurofibromatosis

 

PREFERRED RESPONSE: 3

 

DISCUSSION: Chondrosarcomas associated with diffuse bone lesions (enchondromas) are characteristic of Ollier’s disease.  When accompanied by subcutaneous masses (hemangiomas), the condition is called Maffucci’s syndrome.  Multiple hereditary exostosis is characterized by diffuse osteochondromas.  McCune-Albright syndrome is characterized by polyostotic fibrous dysplasia with cafe-au-lait spots and precocious puberty.  Neurofibromatosis can have associated bone lesions but is not associated with chondrosarcomas.

 

REFERENCES: Sun TC, Swee TC: Chondrosarcoma in Maffucci’s syndrome.  J Bone Joint Surg Am 1985;67:1214-1219.

Schwartz HS, Zimmerman NB, Simon MA, et al: The malignant potential of enchondromatosis.  J Bone Joint Surg Am 1987;69:269-274.

Began WB: Dyschondroplasia and hemangiomata (Maffucci’s syndrome).  Arch Intern Med 1958;102:544.

 

26.     What is the current 5-year survival rate for patients with classic nonmetastatic, high-grade osteosarcoma of the extremity?

 

1-         10%

2-         20%

3-         40%

4-         70%

5-         90%

 

PREFERRED RESPONSE: 4

 

DISCUSSION: Multidisciplinary treatment combining systemic chemotherapy and adequate surgical resection has resulted in a 5-year survival rate of 70% in patients with nonmetastatic osteosarcoma of the extremity .  The advent of effective chemotherapy has increased the overall survival rate from 20% to 70% in current studies. 

 

REFERENCES: Arndt CA, Crist WM: Common musculoskeletal tumors of childhood and adolescence.  N Engl J Med 1999;341:342-352.

Glasser DB, Lane JM, Huvos AG, Marcove RC, Rosen G: Survival, prognosis, and therapeutic response in osteogenic sarcoma: The Memorial Hospital experience.  Cancer 1992;69:698-708.

 

27.     A 43-year-old woman has had pain in the left hip for the past 2 months.  A radiograph, CT scan, MRI scan, and biopsy specimens are shown in Figures 16a through 16e.  What is the most likely diagnosis?

 

1-         Osteosarcoma

2-         Osteochondroma

3-         Chondrosarcoma

4-         Chordoma

5-         Enchondroma

 

PREFERRED RESPONSE: 3

 

DISCUSSION: The imaging studies are consistent with a chondrosarcoma.  The radiograph shows a radiolucent lesion in the pelvis, and there are stippled calcifications on the CT scan.  The histology shows a low-grade cellular hyaline cartilage neoplasm with stellate, occasionally binucleated chondrocytes.  Enchondroma has a more benign histologic appearance. 

 

REFERENCE: Mirra JM, Gold R, Downs J, Eckardt JJ: A new histologic approach to the differentiation of enchondroma and chondrosarcoma of the bones: A clinicopathologic analysis of 51 cases.  Clin Orthop 1985;201:214-237.

 

 

28.     A 40-year-old man with amyloidosis injured his left knee while walking.  Figure 17a shows an AP radiograph that was obtained 2 weeks after the injury.  The radiograph shown in Figure 17b was obtained after the patient wore a hinged knee brace for 3 months.  A clinical photograph is shown in Figure 17c.  What is the most likely diagnosis?

 

1-         Pyarthrosis

2-         Pigmented villonodular synovitis

3-         Synovial osteochondromatosis

4-         Charcot arthropathy

5-         Spontaneous osteonecrosis

 

PREFERRED RESPONSE: 4

 

DISCUSSION: The patient has a Charcot arthropathy of the knee, which is associated with amyloidosis.  The rapid joint destruction shown in the radiographs is most consistent with that diagnosis. 

 

REFERENCES: Drennan D, Fahey J, Maylahn D: Important factors in achieving arthrodesis of the Charcot knee.  J Bone Joint Surg Am 1971;53:1180-1193.

Soudry M, Binazzi R, Johanson N, et al: Total knee arthroplasty in Charcot and Charcot like joints.  Clin Orthop 1986;208:199-204.

Buxbaum JN: The systemic amyloidoses.  Curr Opin Rhemutol 2004;16:67-75.

 

29.     Following preoperative chemotherapy, the percent of tumor necrosis has been shown to be of prognostic value for which of the following tumors?

 

1-         Rhabdomyosarcoma

2-         Chondrosarcoma

3-         Metastatic adenocarcinoma

4-         Osteosarcoma

5-         Giant cell tumor of bone

 

PREFERRED RESPONSE: 4

 

DISCUSSION: The grading of response to chemotherapy for osteosarcoma was introduced by Huvos and associates.  Patients with tumors that show more than 90% necrosis after neoadjuvant chemotherapy are considered to have had a good response and have better survival rates than those with less than 90% necrosis. However, it should be noted that survival rates for patients with a poor response are still better than in patients who do not receive neoadjuvant chemotherapy.  More recently, similar results have been reported in patients with Ewing’s sarcoma.  Chemotherapy is not typically used for giant cell tumor of bone.

 

REFERENCES: Meyers PA, Heller G, Healey J, Huvos A, Lane J, Marcove R, et al: Chemotherapy for nonmetastatic osteogenic sarcoma: The Memorial Sloan- Kettering experience.  J Clin Oncol 1992;10:5-15.

Wunder JS, Paulian G, Huvos AG, Heller G, Meyers PA, Healey JH: The histological response to chemotherapy as a predictor of the oncological outcome of operative treatment of Ewing sarcoma.  J Bone Joint Surg Am 1998;80:1020-1033.

 

30.     Which of the following procedures is not part of the routine evaluation of a patient with suspected metastatic disease to bone?

 

1-         Thorough history and physical examination

2-         CBC count, electrolytes, and serum protein electrophoresis

3-         Bone scan

4-         CT of the brain

5-         Radiograph and CT of the chest

 

PREFERRED RESPONSE: 4

 

DISCUSSION: The work-up for a patient with an unknown primary lesion that is metastatic to bone includes a search for the primary tumor and other sites of metastasis.  This involves conducting a thorough history and physical examination, appropriate laboratory studies, bone scan, chest radiograph, and CT of the chest, abdomen, and pelvis.  In women, a mammogram may be indicated.  CT of the brain is not necessary in the early stages of a metastatic work-up.

 

REFERENCES: Simon MA, Bartucci EJ: The search for the primary tumor in patients with skeletal metastases of unknown origin.  Cancer 1986;58:1088-1095.

Frassica FJ, Gitelis S, Sim FH: Metastic bone disease: General principles, pathophysiology, evaluation, and biopsy.  Instr Course Lect 1992;41:293-300.

 

31.      A 43-year-old man has had right groin pain for the past 3 months.  A radiograph, CT scan, and biopsy specimen are shown in Figures 18a through 18c.  What is the most likely diagnosis?

 

1-         Chordoma

2-         Enchondroma

3-         Dedifferentiated chondrosarcoma

4-         Chondrosarcoma

5-         Metastatic renal cell carcinoma 

 

PREFERRED RESPONSE: 4

 

DISCUSSION: The radiographic appearance of the lesion is lytic with calcifications in the supra-acetabular region.  The CT scan shows bone destruction and periosteal reaction, which rules out an enchondroma.  The histologic appearance is that of cellular cartilage, with a high cytoplasmic:nuclear ratio and basophilic appearance to the cytoplasm; these findings rule out chordoma and metastatic renal cell carcinoma.  In addition, chordomas are most frequently found in the sacrum and base of the skull.  Because there is no high-grade spindle cell component to suggest dedifferentiation, the most likely diagnosis is chondrosarcoma.

 

REFERENCES: Pring M, Weber KL, Unni K, Sim FH: Chondrosarcoma of the pelvis: A review of sixty-four cases.  J Bone Joint Surg Am 2001;83:1630-1642.

Unni KK: Dahlin’s Bone Tumors: General Aspects and Data on 11,087 Cases, ed 5.  Philadelphia, PA, Lippincott-Raven, 1996, pp 71-92.

 

32.     What malignant disease most commonly develops in conjunction with chronic osteomyelitis?

 

1-         Fibrosarcoma

2-         Basal cell carcinoma

3-         Lymphoma

4-         Osteosarcoma

5-         Squamous cell carcinoma

 

PREFERRED RESPONSE: 5

 

DISCUSSION: The most common malignant disease to arise in conjunction with chronic osteomyelitis is squamous cell carcinoma particularly in patients with a long-standing draining sinus tract.

 

REFERENCES: Dell PC: Hand, in Simon MA, Springfield D (eds): Surgery for Bone and Soft Tissue Tumors.  Philadelphia, PA, Lippincott-Raven, 1998, pp 405-420.

McGrory JE, Pritchard DJ, Unni KK, Ilstrup D, Rowland CM: Malignant lesion arising in chronic osteomyelitis.  Clin Orthop 1998;362:181-189.

 

33.       A 77-year-old man has had increasing right knee pain for the past 3 months. A radiograph and coronal T1-weighted MRI scan are shown in Figures 19a and 19b.  A biopsy specimen is shown in Figure 19c.  What is the most likely diagnosis?

 

1-         Metastatic prostate cancer

2-         Enchondroma

3-         Osteomyelitis

4-         Dedifferentiated chondrosarcoma

5-         Lymphoma

 

PREFERRED RESPONSE: 4

 

DISCUSSION: The radiograph shows a calcified lesion in the medullary canal of the distal femoral diaphysis.  The MRI scan shows extensive marrow change distal to the lesion, which is not consistent with an enchondroma.  The histology shows a biphasic pattern with low-grade cartilage just apposed to high-grade spindle cell sarcoma.  The overall appearance is consistent with dedifferentiated chondrosarcoma.  The radiographic appearance is not consistent with enchondroma, and the histologic appearance is not consistent with the other choices.

 

REFERENCES: Mitchell AD, Ayoub K, Mangham DC, et al: Experience in the treatment of dedifferentiated chondrosarcoma.  J Bone Joint Surg Br 2000;82:55-61.

Frassica FJ, Unni KK, Beabout JW, Sim FH: Dedifferentiated chondrosarcoma: A report of the clinicopathological features and treatment of seventy-eight cases.  J Bone Joint Surg Am 1986;68:1197-1205.

 

34.     A 10-year-old girl reports activity-related bilateral arm pain.  Examination reveals no soft-tissue masses in either arm, and she has full painless range of motion in both shoulders and elbows.  The radiograph and bone scan are shown in Figures 20a and 20b, and biopsy specimens are shown in Figures 20c and 20d.  What is the most likely diagnosis?

 

1-         Enchondroma

2-         Fibrous dysplasia

3-         Osteogenic sarcoma

4-         Aneurysmal bone cyst

5-         Periosteal chondroma

 

PREFERRED RESPONSE: 2

 

DISCUSSION: Based on these findings, the most likely diagnosis is fibrous dysplasia.  Twenty percent of patients with fibrous dysplasia have multifocal disease.  The lesions show a typical ground glass appearance.  Fibrous dysplasia frequently involves the diaphysis of the long bones.  There is no associated soft-tissue mass and no periosteal reactions to these lesions, suggesting a benign lesion.  The histology shows proliferating fibroblasts in a dense collagen matrix.  Trabeculae are arranged in an irregular or “Chinese letter” appearance.  Osteogenic sarcoma and Ewing’s sarcoma have a much different radiographic appearance of malignant osteoid and small round blue cells.  Periosteal chondroma does occur in the proximal humerus but is not typically multifocal.  It appears as a surface lesion with saucerization of the underlying bone and a bony buttress adjacent to the lesion.  Some patients with multifocal lesions have associated endocrine abnormalities (McCune-Albright syndrome).

 

REFERENCES: Wold LA, et al: Atlas of Orthopaedic Pathology.  Philadelphia, PA, WB Saunders, 1990, pp 118-119.

Simon M, et al: Surgery for Bone and Soft Tissue Tumors.  Philadelphia, PA, Lippincott Raven, 1998, p 197.

 

35.     After excising a mass from the thigh that was thought to be a lipoma, the pathology reveals that the mass is a high-grade sarcoma. Subsequent treatment should include

 

1-         repeat excision of the tumor bed.

2-         chemotherapy only.

3-         radiation therapy only.

4-         chemotherapy and radiation therapy only.

5-         administration of bisphosphonates.

 

PREFERRED RESPONSE: 1

 

DISCUSSION: Following excision of a suspected benign soft-tissue tumor that proves to be malignant, repeat excision of the tumor bed is recommended.  The initial surgical margins are inadequate after an intralesional or marginal excision, necessitating additional surgery for more definitive local control.  While radiation therapy and/or chemotherapy may help to reduce the risk of local recurrence in patients with microscopic residual disease, local control is improved following repeat excision.  Radiation therapy alone is inadequate to address poor surgical margins, and would likely be given postoperatively.  Bisphosphonates have no current role in the treatment of soft-tissue sarcoma.

 

REFERENCES: Noria S, Davis A, Kandel R, et al: Residual disease following unplanned excision of soft-tissue sarcoma of an extremity.  J Bone Joint Surg Am 1996;78:650-655.

Bell RS, O’Sullivan B, Liu FF, et al: The surgical margin in soft-tissue sarcoma.  J Bone Joint Surg Am 1989;71:370-375.

 

36.     Chemotherapy is routinely included in the treatment of which of the following soft-tissue sarcomas?

 

1-         Angiosarcoma

2-         Malignant fibrous histiocytoma

3-         Liposarcoma

4-         Rhabdomyosarcoma

5-         Clear cell sarcoma

 

PREFERRED RESPONSE: 4

 

DISCUSSION: Most soft-tissue sarcomas are treated with a combination of radiation therapy and wide resection.  Rhabdomyosarcomas are an exception, where chemotherapy is included in all treatment plans.  Chemotherapy for other soft-tissue sarcomas is controversial.

 

REFERENCES: Enzinger FM, Weiss SW: Rhabdomyosarcoma, in Soft Tissue Tumors, ed 3.  St Louis, MO, CV Mosby, 1995, p 539.

Hays DM: Rhabdomyosarcoma.  Clin Orthop 1993;289:36-49.

 

37.      A 10-year-old child reports acute leg pain after wrestling with his brother.  AP and lateral radiographs are shown in Figures 21a and 21b.  What is the best course of action?

 

1-         Biopsy, curettage, and plating

2-         Wide segmental resection

3-         Hip disarticulation

4-         Closed reduction and a long leg cast

5-         Tibial traction and MRI

 

PREFERRED RESPONSE: 4

 

DISCUSSION: The radiographs show an eccentric metaphyseal lesion with a well-defined reactive rim of bone that is consistent with a nonossifying fibroma.  Pathologic fractures through benign lesions should be treated as appropriate for the fracture, allowing the fracture to heal.  Biopsy is not needed when the radiographic diagnosis is benign.  MRI, in the presence of a fracture, is not particularly helpful because of the hematoma.  If radiographic findings reveal that the lesion appears aggressive, a biopsy should be performed, obtaining tissue away from the fracture site.

 

REFERENCES: Marks KE, Bauer TW: Fibrous tumors of bone.  Orthop Clin North Am 1989;20:377. 

Ponseti IV, Friedman B: Evaluation of metaphyseal fibrous defects.  J Bone Joint Surg Am 1949;31:582.

 

38.     An 83-year-old man has a painful mass of the great toe.  Radiographs and a biopsy specimen are seen in Figures 22a and 22b.  What is the most likely diagnosis?

 

1-         Gout

2-         Pseudogout

3-         Infection

4-         Epidermal inclusion cyst

5-         Charcot joint

 

PREFERRED RESPONSE: 1

 

DISCUSSION: Gouty arthritis, pseudogout, and infection can all present with inflammatory arthritis and periarticular erosions.  Strongly negative birefringent crystals are seen in gout.  The histologic image shows elongated “needle-like” crystals of gout.  Epidermal inclusion cysts are rarely painful and usually have a history of localized penetrating trauma. 

 

REFERENCES: Hamilton W, Breedman KB, Haupt HM, Lackman R: Knee pain in a 40-year-old man.  Clin Orthop 2001;383:282-285,290-292.

Mizel M, Miller R, Scioli M (eds): Orthopaedic Knowledge Update: Foot and Ankle 2.  Rosemont, IL, American Academy of Orthopaedic Surgeons, 1998, pp 301-302.

 

39.     Eosinophilic granuloma frequently occurs as a solitary lesion in the tubular long bones.  After biopsy, what is the best course of action?

 

1-         Neoadjuvant chemotherapy

2-         En bloc resection

3-         Observation

4-         Amputation

5-         Chemotherapy followed by radiation therapy

 

PREFERRED RESPONSE: 3

 

DISCUSSION: Most lesions of eosinophilic granuloma are simply observed, but larger aggressive lesions may require curettage and bone grafting.  Frequently, biopsy is required to rule out a malignant diagnosis.  The differential diagnosis of eosinophilic granuloma is osteomyelitis, Ewing’s sarcoma of bone, or osteogenic sarcoma.  The biopsy alone can be followed by spontaneous resolution.  In some patients, low-dose radiation therapy is used.  Chemotherapy or amputation is not indicated for these benign lesions.

 

REFERENCE: Simon M, Springfield D, et al: Common Benign Bone Tumors: Surgery for Bone and Soft Tissue Tumors.  Philadelphia, PA, Lippincott Raven, 1998, p 200.

 

40.     A 52-year-old man has had back pain radiating to the left leg for the past 5 weeks.  A radiograph, MRI scans, and biopsy specimens are shown in Figures 23a through 23f.  What is the most likely diagnosis?

 

1-         Metastatic renal cell carcinoma

2-         Metastatic adenocarcinoma

3-         Chordoma

4-         Osteoblastoma

5-         Chondrosarcoma

 

PREFERRED RESPONSE: 3

 

DISCUSSION: The histology shows cells with bubbly, abundant clear cytoplasm typical of physaliphorous cells; therefore, the most likely diagnosis is chordoma.  These tumors arise from notocord rests in the upper and lower spine.

 

REFERENCE: Mirra J: Bone Tumors: Clinical, Radiologic, and Pathologic Correlations.  Philadelphia, PA, Lea and Febiger, 1989, vol 1, ch 8.

 

41.      A 15-year-old boy has had pain in the right knee for the past 3 months.  He denies any history of trauma.  Examination reveals a firm mass in the distal thigh; the remainder of the examination is unremarkable.  A radiograph is shown in Figure 24.  What further work-up should be completed prior to biopsy?

 

1-         None

2-         MRI of the distal femur

3-         MRI of the entire femur

4-         MRI of the entire femur and a bone scan

5-         MRI of the entire femur, CT of the chest, and a bone scan

 

PREFERRED RESPONSE: 5

 

DISCUSSION: The radiograph shows an aggressive destructive lesion.  In this age group, and based on the anatomic location, a primary malignant tumor (osteosarcoma) is likely.  Additional staging studies to identify metastatic disease are imperative prior to any biopsy.  MRI of the femur helps to reveal skip metastasis and provides information regarding the anatomic location of the lesion.  CT of the chest and a bone scan evaluate for distant metastatic spread.  A bone scan is also useful in evaluating the extent of local bone activity about the lesion.

 

REFERENCES: Simon MA, Finn HA: Diagnostic strategy for bone and soft tissue tumors.  J Bone Joint Surg Am 1993;75:622-631.

Simon M, Springfield D, et al: Biopsy: Surgery for Bone and Soft Tissue Tumors.  Philadelphia, PA, Lippincott Raven, 1998, p 6.

 

42.     What is the most common primary malignant tumor of bone in childhood?

 

1-         Periosteal chondroma

2-         Ewing’s sarcoma

3-         Osteosarcoma

4-         Chondrosarcoma

5-         Rhabdomyosarcoma

 

PREFERRED RESPONSE: 3

 

DISCUSSION: Osteosarcoma is the most common primary malignant tumor of bone in childhood, followed by Ewing’s sarcoma.  Rhabdomyosarcoma is a soft-tissue sarcoma of childhood.  Chondrosarcoma rarely occurs in childhood.  Osteochondromas are benign tumors of the bone.

 

REFERENCES: Simon M, Springfield D, et al: Osteogenic Sarcoma: Surgery for Bone and Soft Tissue Tumors.  Philadelphia, PA, Lippincott Raven, 1998, p 226. 

Wold LA, et al: Osteogenic Sarcoma: Atlas of Orthopaedic Pathology.  Philadelphia, PA, WB Saunders, 1990, pp 14-15.

 

43.     A patient with a 5-cm synovial sarcoma located in the distal portion of the rectus femoris muscle undergoes excision of the mass.  The procedure is performed through a 10-cm longitudinal incision.  Only a portion of the rectus femoris is removed; the vast majority of the muscle is preserved.  The plane of dissection is beyond the reactive zone, and the pathology reveals that the margins are negative.  This procedure is classified as

 

1-         radical.

2-         marginal.

3-         wide.

4-         intralesional.

5-         within the reactive zone.

 

PREFERRED RESPONSE: 3

 

DISCUSSION: The patient underwent a wide resection, which involves excision of the tumor along with a cuff of normal tissue that completely surrounds the tumor.  The plane of resection is beyond the reactive zone.  A radical resection involves removal of the entire affected muscle from origin to insertion.  In a marginal excision, the plane of dissection is through the reactive zone of the tumor.  A marginal excision is generally considered inadequate surgery for high-grade sarcomas.  In an intralesional resection, the plane of dissection is through the tumor.  Excision within the reactive zone but beyond the tumor is the same as a marginal excision.

 

REFERENCES: Enneking WF: Staging of musculoskeletal neoplasms, in Current Concepts of Diagnosis and Treatment of Bone and Soft Tissue Tumors.  Heidelberg, Germany, Springer-Verlag, 1984.

Simon MA, Springfield D: Surgery for Bone and Soft Tissue Tumors.  Philadelphia, PA, Lippincott-Raven, 1998.

 

44.     Which of the following tumors is most likely to present with a pathologic fracture in a child?

 

1-         Unicameral bone cyst

2-         Fibrous cortical defect

3-         Osteosarcoma

4-         Ewing’s sarcoma

5-         Giant cell tumor

 

PREFERRED RESPONSE: 1

 

DISCUSSION: In nearly 50% of patients with a unicameral bone cyst, the lesion remains asymptomatic until a fracture occurs, usually as the result of relatively minor trauma.  If the lesion expands, the bone is weakened and may cause pain.  Fibrous cortical defects are usually an incidental finding and typically asymptomatic.  Malignant bone tumors such as osteosarcoma and Ewing’s sarcoma most commonly cause pain, and pathologic fracture occurs in less than 10% of patients.  Giant cell tumors are uncommon in children and usually are painful.

 

REFERENCES: Wilkins RM: Unicameral bone cysts.  J Am Acad Orthop Surg 2000;8:217-224.

Dormans JP, Pill SG: Fractures through bone cysts: Unicameral bone cysts, aneurysmal bone cysts, fibrous cortical defects, and nonossifying fibromas.  Instr Course Lect 2002;51:457-467.

Hecht AC, Gebhardt MC: Diagnosis and treatment of unicameral and aneurysmal bone cysts in children.  Curr Opin Pediatr 1998;10:87-94.

 

45.     A 60-year-old woman has a mass in the right scapula.  Figures 25a and 25b show a CT scan and a biopsy specimen.  The cells are lymphocyte common antigen positive, Ewing’s specific antigen (CD99) negative, and keratin negative.  What is the next step in management?

 

1-         Skeletal survey

2-         PET scan

3-         Indium-labeled WBC scan

4-         Bone marrow aspiration and biopsy

5-         Mammography

 

PREFERRED RESPONSE: 4

 

DISCUSSION: The clinical history, CT scan, and histology are most consistent with a lymphoma of bone.  An important part of the staging is bone marrow aspiration and biopsy.  The other studies listed are not indicated.  Lymphoma of bone, when localized, is usually treated with chemotherapy and radiation therapy and has excellent survival rates.  Widespread lymphoma has a worse prognosis.

 

REFERENCES: Finiewicz K, van Biesen K: Non-Hodgkins lymphoma, in Golomb H, Vokes E (eds): Oncologic Therapies, ed 2.  Berlin, Germany, Springer, 2003, pp 295-318.

Lems P, Primus G, Anastas J, Doherty D, Montag AG, Peabody TD, Simon MA: Oncologic outcomes of primary lymphoma of bone in adults.  Clin Orthop 2003;415:90-97.

 

46.     The use of multiagent adjuvant chemotherapy is associated with a clear survival benefit in which of the following diseases?

 

1-         Renal carcinoma

2-         Osteosarcoma

3-         Dedifferentiated chondrosarcoma

4-         Adult soft-tissue sarcoma

5-         Melanoma

 

PREFERRED RESPONSE: 2

 

DISCUSSION: The use of multiagent chemotherapy has been shown to be associated with a survival benefit in patients with osteosarcoma.  The use of chemotherapy in adults with soft-tissue sarcoma remains somewhat controversial.  It has not been associated with improved survival rates in patients with renal carcinoma, dedifferentiated chondrosarcoma, or melanoma. 

 

REFERENCES: Menendez LR (ed): Orthopaedic Knowledge Update: Musculoskeletal Tumors.  Rosemont, IL, American Academy of Orthopaedic Surgeons, 2002, p 53.

Link M, Goorin A, Miser A, et al: The effect of adjuvant chemotherapy and relapse free survival in patients with osteosarcoma of the extremity.  N Engl J Med 1986;314:1600-1606.

 

47.      Which of the following definitions best describes Batson’s vertebral vein system?

 

1-         A system of valves that allows retrograde embolism

2-         A vascular system that bypasses organs such as the breast, prostate, lung, kidney, and thyroid

3-         A system of veins within the thoracoabdominal cavity that are not subject to compression or collapse by increases in intra-abdominal pressure

4-         A series of valveless veins that allow tumor cell extravasation

5-         A longitudinal vein system with a segmental distribution to the vertebra that extends from the dural venous sinuses of the skull to the sacrum

 

PREFERRED RESPONSE: 4

 

DISCUSSION: The venous plexus was described by Batson and helps to explain the common distribution of metastatic cells to the vertebrae, skull, ribs, and proximal long bones.  Batson studied the vertebral vein system extensively by using contrast agents in human cadavers and live monkeys.  Batson’s plexus is a valveless system that allows retrograde embolism from the major organs such as the breast, prostate, lung, kidney, and thyroid.  It is located within the thoracoabdominal cavity and has connections to the proximal long bones and an intercommunicating network of thin-walled veins with a low intraluminal pressure.

 

REFERENCES: Batson OV: Function of vertebral veins and their role in spread of metastases.  Ann Surg 1940;112:138-149.

Coman DR, de Long RP: Role of vertebral venous system in metastasis of cancer to spinal column: Experiments with tumor-cell suspensions in rats and rabbits.  Cancer 1951;4:610-618.

 

48.     Figure 26 shows the radiograph of a 48-year-old woman who has right arm pain and hematuria.  A bone scan reveals increased uptake in the left ribs and thoracic spine.  A needle biopsy specimen shows that the lesion is highly keratin positive and composed primarily of clear cells.  What is the best course of action?

 

1-         Interleukin-2-based chemotherapy

2-         Embolization followed by curettage, intramedullary fixation, and cementation

3-         Embolization followed by curettage and cementation

4-         External beam radiation

5-         Total humeral resection and prosthetic reconstruction

 

PREFERRED RESPONSE: 2

 

DISCUSSION: The lesion has the typical “blown out” lytic radiographic appearance that is most commonly found in thyroid or renal cell metastases.  Given the history of hematuria and histology findings, the most likely diagnosis is metastatic renal cell carcinoma.  This tumor is relatively resistant to chemotherapy.   Radiation therapy is used as a postoperative adjuvant treatment with varying response rates.  Surgery should be performed after preoperative embolization to decrease the risk of intraoperative bleeding, as no tourniquet can be used in this location.  Patients with metastatic renal cell carcinomas may survive for years, resulting in a higher likelihood of local tumor progression with ineffective adjuvant therapy.  Intramedullary fixation combined with curettage and cementation will provide the best chance of local control while maintaining the patient’s native shoulder and elbow joints.  A total humeral resection is an extensive surgery with considerable morbidity and is not indicated for this patient because less extensive surgery is likely to be effective.

 

REFERENCES: Harrington KD, Sim FH, Enis JE, Johnston JO, Diok HM, Gristina AG: Methylmethacrylate as an adjunct in internal fixation of pathological fractures: Experience with three hundred and seventy-five cases.  J Bone Joint Surg Am 1976;58:1047-1054.

Sun S, Lang EV: Bone metastases from renal cell carcinoma: Preoperative embolization.  J Vasc Interv Radiol 1998;9:263-269.

Katzner M, Schvingt E: Operative treatment of bone metastases secondary to renal carcinoma: Basic research and treatment of renal cell carcinoma metastasis.  Prog Clin Biol Res EORTC 1990;348:151-168.

 

49.     A 15-year-old boy has had pain in the right shoulder for the past 3 months.  He denies any history of trauma and has no constitutional symptoms.  Examination reveals a large firm mass in the proximal arm.  A radiograph and MRI scan are shown in Figures 27a and 27b.  Biopsy specimens are shown in Figures 27c and 27d.  Management should consist of

 

1-         observation.

2-         steroid injection.

3-         curettage and bone grafting.

4-         wide resection with neoadjuvant chemotherapy.

5-         debridement, irrigation, and intravenous antibiotics.

 

PREFERRED RESPONSE: 3

 

DISCUSSION: The patient has an aneurysmal bone cyst.  The fluid-fluid levels seen on the MRI scan are typical for aneurysmal bone cyst, and the  histology is consistent with a cystic lining.  Vascular lakes, multinucleated giant cells, reactive bone, fibrovascular tissue, and an absence of atypical cells or numerous mitoses are seen histologically.  Aneurysmal bone cysts will typically continue to grow and cause further bone destruction; therefore, observation is not recommended.  Steroid injections are not effective.  A thorough curettage of the cyst lining and bone grafting are required.  Wide resection and chemotherapy are reserved for more aggressive tumors.  There is no evidence of infection radiographically or histologically.  Telangiectatic osteosarcoma should also be considered in the differential diagnosis; therefore, biopsy is an important part of the work-up. 

 

REFERENCES: Wold LA, et al: Atlas of Orthopaedic Pathology.  Philadelphia, PA, WB Saunders, 1990, pp 232-233.

Simon M, et al: Surgery for Bone and Soft Tissue Tumors.  Philadelphia, PA, Lippincott Raven, 1998, pp 194-196.

 

50.       A 38-year-old woman with metastatic thyroid carcinoma has had increasing pain in the left hip for the past 3 months.  An AP radiograph and coronal T1-weighted MRI scan are shown in Figures 28a and 28b.  Management should consist of

 

1-         external beam radiation.

2-         curettage and cementation of the lesion with a compression hip screw and side plate fixation.

3-         curettage and cementation of the lesion with intramedullary fixation.

4-         cemented bipolar hemiarthroplasty.

5-         radioactive iodine infusion.

 

PREFERRED RESPONSE: 4

 

DISCUSSION: The radiograph and MRI scan reveal a lytic lesion in the left femoral neck region that extends to the lesser trochanter.  Although external beam radiation and radioactive iodine infusion may be helpful in controlling the local disease, the patient is at high risk for femoral neck fracture given the location of the lesion.  Prophylactic surgery is indicated; therefore, the treatment of choice is a cemented bipolar hemiarthroplasty.  The use of a compression hip screw and side plate or an intramedullary nail has a high likelihood of failure with disease progression.  Postoperative treatment with radiation therapy and bisphosphonates is also indicated.

 

REFERENCES: Mirels H: Metastatic disease in long bones: A proposed scoring system for diagnosing impending pathologic fractures. Clin Orthop 1989;249:256-264.

Swanson KC, Pritchard DJ, Sim FH: Surgical treatment of metastatic disease of the femur.  J Am Acad Orthop Surg 2000;8:56-65.

Clarke HD, Damron TA, Sim FH: Head and neck replacement endoprosthesis for pathologic proximal femoral lesions.  Clin Orthop 1998;353:210-217.

 

51.      What is the most common location of osteosarcoma?

 

1-         Shoulder

2-         Spine

3-         Pelvis

4-         Knee

5-         Skull

 

PREFERRED RESPONSE: 4

 

DISCUSSION: The most common location of osteosarcoma is the knee area (50% to 55%), followed by the proximal humerus and iliac wing.  The most commonly involved long bone is the femur (40% to 45%), followed by the tibia (15% to 25%).  Within these bones, tumors are typically adjacent to the epiphyses in most patients.  The flat bones of the pelvis and spine are less frequently involved.

 

REFERENCES: Malawer MM, Sugarbaker PH, Malawer M: Musculoskeletal Cancer Surgery: Treatment of Sarcomas and Allied Diseases.  Kluwer Academic Publishers, 2001.

Wold LA, et al:  Osteogenic Sarcoma: Atlas of Orthopedic Pathology.  Philadelphia, PA, WB Saunders, 1990, pp 14-15.

 

52.     A 40-year-old man with an acetabular chondrosarcoma has a small soft-tissue mass.  Treatment should consist of

 

1-         chemotherapy with ifosfamide and doxorubicin hydrochloride.

2-         external beam radiation.

3-         wide resection.

4-         curettage and cementation of the pelvic lesion.

5-         cemented total hip arthroplasty with acetabular reconstruction.

 

PREFERRED RESPONSE: 3

 

DISCUSSION: The treatment of choice for pelvic chondrosarcoma is wide resection via an internal hemipelvectomy.  Chondrosarcoma requires surgical resection for control and does not respond to traditional chemotherapy or external beam radiation.  Hip arthroplasty with acetabular reconstruction and curettage and cementation of the lesion are intralesional procedures that result in a higher incidence of local recurrence of tumor. 

 

REFERENCES: Pring M, Weber KL, Unni K, Sim FH: Chondrosarcoma of the pelvis: A review of sixty-four cases.  J Bone Joint Surg Am 2001;83:1630-1642.

Sheth DS, Yasko AW, Johnson ME, Ayala AG, Murray JA, Romsdahl MM: Chondrosarcoma of the pelvis: Prognostic factors for 67 patients treated with definitive surgery.  Cancer 1996;78:745-750.

 

53.     Figures 29a and 29b show the AP radiograph and CT scan of a 70-year-old man who has left thigh pain.  Serum protein electrophoresis shows a monoclonal gammopathy.  Additional radiographs of the femur show other lesions.  Management should consist of

 

1-         chemotherapy, wide resection, and endoprosthetic reconstruction.

2-         radiation therapy only.

3-         chemotherapy only.

4-         prophylactic internal fixation with a locked intramedullary rod.

5-         open curettage, bone grafting, dynamic hip screw fixation, and radiation therapy.

 

 

PREFERRED RESPONSE: 4

 

DISCUSSION: The underlying diagnosis is multiple myeloma. Because the patient has a large lucent lesion in the peritrochanteric region of the left proximal femur, the risk of pathologic fracture is high.  Consideration should be given to prophylactic internal fixation with a locked intramedullary rod.  The lesion does not appear to be a sarcoma requiring wide resection and endoprosthetic reconstruction.  Neither chemotherapy nor radiation therapy alone is likely to result in long-term stabilization of the proximal femur.  Postoperative treatment with bisphosphonates and radiation therapy is indicated to decrease the risk of future pathologic fractures.  The patient should also be referred to a medical oncologist for medical management.

 

REFERENCES: Menendez LR (ed): Orthopaedic Knowledge Update: Musculoskeletal Tumors.  Rosemont, IL, American Academy of Orthopaedic Surgeons, 2002, p 364.

Mirels H: Metastatic disease in long bones: A proposed scoring system for diagnosing impending pathologic fractures.  Clin Orthop 1989;249:256-264.

 

54.     What pharmacologic agents are preferred for the treatment of symptomatic active Paget’s disease?

 

1-         Nasal calcitonin

2-         Bisphosphonates

3-         Nonsteroidal anti-inflammatory drugs

4-         Furosemide

5-         Antiviral therapy

 

PREFERRED RESPONSE: 2

 

DISCUSSION: Recent medical literature supports the use of bisphosphonates as the treatment of choice for active Paget’s disease.

 

REFERENCE: Delman PD, Meunier PJ: The management of Paget’s disease.  N Eng J Med 1997;336:558-566.

 

55.     A 7-year-old girl has pain and a mass in the left scapula.  A MRI scan and biopsy specimen are shown in Figures 30a and 30b.  After staging studies, initial management should consist of

 

1-         radiation therapy to the scapula.

2-         systemic chemotherapy.

3-         total scapulectomy.

4-         partial scapulectomy.

5-         intravenous antibiotics.

 

PREFERRED RESPONSE: 2

 

DISCUSSION: The histology shows small round blue cells that are uniform in appearance; these findings are consistent with Ewing’s sarcoma.  The MRI scan shows infiltration of the marrow and a large surrounding soft-tissue mass.  Based on these findings, the management of choice is systemic chemotherapy.  Local control of the primary lesion is addressed by either surgical resection or radiation therapy or a combination of the two after the patient receives systemic chemotherapy.  The clinical, radiographic, and histologic presentation of Ewing’s sarcoma often can be confused with osteomyelitis.  The histology shows an absence of inflammatory cells.

 

REFERENCES: Grier HE: The Ewing family of tumors: Ewing’s sarcoma and primitive neuroectodermal tumors.  Pediatr Clin North Am 1997;44:991-1004.

Arndt CA, Crist WM: Common musculoskeletal tumors of childhood and adolescence.  N Engl J Med 1999;341:342-352.

 

56.     A 73-year-old man reports increasing back and lower extremity pain.  A bone scan is shown in Figure 31.  What is the most likely diagnosis?

 

1-         Multiple myeloma

2-         Metastatic neuroblastoma

3-         Polyostotic fibrous dysplasia

4-         Hodgkin’s lymphoma

5-         Metastatic prostate cancer

 

PREFERRED RESPONSE: 5

 

DISCUSSION: The bone scan reveals lesions throughout the skeleton.  The patient’s age, gender, and pain pattern are consistent with metastatic prostate cancer.  Multiple myeloma typically does not have enough osteoblastic activity to produce this bone scan.  The patient’s age is not consistent with metastatic neuroblastoma (a pediatric disease).  Polyostotic fibrous dysplasia may involve multiple active lesions in younger patients but does not have such a widespread distribution of lesions.  Hodgkin’s lymphoma can involve bone, but the widespread discrete appearance on this bone scan is most consistent with metastatic prostate cancer.  In a patient with widespread bone metastases from prostate cancer, bisphosphonates may play a critical role in treatment by decreasing pain and the number of fractures.

 

REFERENCES: Roudier MP, Vesselle H, True LD, Higano CS, Ott SM, King SH, Vessella RL: Bone histology at autopsy and matched bone scintigraphy findings in patients with hormone refractory prostate cancer: The effect of bisphosphonate therapy on bone scintigraphy results.  Clin Exp Metastasis 2003;20:171-180.

Sartor O: Radioisotopic treatment of bone pain from metastatic prostate cancer.  Curr Oncol Rep 2003;5:258-262.

 

57.      A 16-year-old girl has had pain in the left groin for the past 4 months.  She notes that the pain is worse at night; however, she denies any history of trauma and has no constitutional symptoms.  There is no history of steroid or alcohol use.  Examination reveals pain in the left groin with rotation of the hip.  There is no associated soft-tissue mass.  A radiograph and MRI scan are shown in Figures 32a and 32b, and biopsy specimens are shown in Figures 32c and 32d.  What is the most likely diagnosis?

 

1-         Clear cell chondrosarcoma

2-         Chondroblastoma

3-         Giant cell tumor

4-         Aneurysmal bone cyst

5-         Osteonecrosis of the femoral head

 

PREFERRED RESPONSE: 2

 

DISCUSSION: Based on the epiphyseal location and sharp, well-defined borders, the radiograph suggests chondroblastoma.  Histologically, multinucleated giant cells are scattered among mononuclear cells.  The nuclei are homogenous and contain a characteristic longitudinal groove.  Although not seen here, “chicken-wire calcification” with a bland giant cell-rich matrix is also typical for chondroblastoma.  Clear cell chondrosarcoma occurs in epiphyseal locations but has a more aggressive histologic pattern and occurs in an older age group.  Giant cell tumors occur in the epiphysis but have a more uniform giant cell population histologically.  Aneurysmal bone cyst often results in bone remodeling and has a different pathologic appearance.  Osteonecrosis has a typical histologic pattern of empty lacunae and necrotic bone.

 

REFERENCES: Springfield DS, Capanna R, Gherlinzoni F, et al: Chondroblastoma: A review of seventy cases.  J Bone Joint Surg Am 1985;67:748-755.

Simon M, Springfield D, et al: Chrondroblastoma: Surgery for Bone and Soft Tissue Tumors. Philadelphia, PA, Lippincott Raven, 1998, p 190.

Wold LA, et al: Atlas of Orthopaedic Pathology.  Philadelphia, PA, WB Saunders, 1990,

pp 62-67.

 

58.     Ewing’s sarcoma of bone most commonly occurs in which of the following locations?

 

1-         Major long tubular bones

2-         Flat bones of the pelvis

3-         Fibula

4-         Metacarpals

5-         Vertebral bodies

 

PREFERRED RESPONSE: 1

 

DISCUSSION: Ewing’s sarcoma typically occurs in the major long tubular bones, with the femur the most common location.  The flat bones of the pelvis are the second most common location.  Ewing’s sarcoma occurs in the fibula but with a lower incidence than that seen in the major tubular bones.  Ewing’s sarcoma infrequently occurs in the metacarpals or the vertebral bodies.

 

REFERENCES: Simon M, Springfield D, et al: Ewing’s Sarcoma: Surgery for Bone and Soft Tissue Tumors.  Philadelphia, PA, Lippincott Raven, 1998, p 287.

Wold LA, et al: Ewing’s Sarcoma: Atlas of Orthopaedic Pathology.  Philadelphia, PA, WB Saunders, 1990, pp 210-211.

 

59.     A previously healthy 14-year-old boy now reports fatigue, and has a bilateral Trendelenburg gait, right hip pain, and bilateral knee and foot pain.  Biopsy of a right sacral mass reveals intermediate grade osteosarcoma.  There are no metastases.  Laboratory studies reveal a serum calcium level of 7.7 mg/dL (normal 8.5 to 10.5), a phosphate level of 2.0 mg/dL (normal 2.7 to 4.5), a 1,25-dihydroxyvitamin D level of less than 10 pg/mL (normal 18 to 62), a parathyroid hormone level of 19 pg/mL (normal 10 to 60), and an alkaline phosphatase level of 428 U/L (normal 15 to 351).  What is the most likely cause of the patient’s symptoms?

 

1-         Oncogenic rickets

2-         Calcium sequestration by the tumor

3-         Elevated alkaline phosphatase level

4-         Tumor cachexia

5-         L5 neuropathy

 

PREFERRED RESPONSE: 1

 

DISCUSSION: The laboratory findings are typical for rickets.  Oncogenic rickets is a paraneoplastic syndrome that results from a substance secreted by the tumor that interferes with renal tubule reabsorption of phosphate.  This substance previously had been called phosphatonin but recently has been identified as fibroblast growth factor 23.  Nutritional rickets is rare in developed countries.  Delayed onset familial hypophosphatemic rickets is possible, but the likelihood of having two rare diseases is unlikely.  Osteosarcoma does not sequester calcium.  Alkaline phosphatase levels can be elevated in osteosarcoma but does not cause muscle weakness.  Tumor cachexia would occur only with advanced metastatic disease.  A unilateral sacral mass would not cause a bilateral L5 neuropathy or the abnormal laboratory findings.

 

REFERENCES: Case records of the Massachusetts General Hospital.  Weekly clinicopathological exercises.  Case 29-2001.  A 14-year-old with abnormal bones and a sacral mass.  N Engl J Med 2001;345:903-908.

Jonsson KB, Zahradnik R, Larsson T, White KE, Sugimoto T, Imanishi Y, et al: Fibroblast growth factor 23 in oncogenic osteomalacia and X-linked hypophosphatemia.  N Engl J Med 2003;348:1656-1663.

 

60.     Which of the following staging studies should be obtained for an adult with an 8-cm deep, high-grade malignant fibrous histiocytoma of the extremity?

 

1-         MRI of the lesion, CT of the chest

2-         MRI of the lesion, chest, and abdomen, CT of the pelvis

3-         MRI of the lesion, chest, and abdomen, CT of the pelvis, bone scan

4-         MRI of the lesion, CT of the chest, sentinel node biopsy

5-         CT of the lesion and chest, bone scan

 

PREFERRED RESPONSE: 1

 

DISCUSSION: MRI is the preferred imaging study to evaluate the local tumor extension for soft-tissue lesions, but CT can be used if MRI is contraindicated (eg, patients with pacemakers).  CT of the chest is always recommended in patients with high-grade sarcomas because 80% of metastases occur in the lungs.  CT of the abdomen and pelvis is indicated in patients with lower extremity liposarcoma because some patients also have synchronous retroperitoneal liposarcoma.  Lymph node metastasis occurs in up to 5% of patients with soft-tissue sarcoma.  If the nodes are clinically enlarged, biopsy is indicated.  Routine sentinel node biopsy currently is not recommended.  Bone scan is not used in the staging of soft-tissue sarcoma as it has not been shown to be cost-effective.

 

REFERENCES: Demetri GD, Pollock R, Baker L, Balcerzak S, Casper E, Conrad C, et al: NCCN sarcoma practice guidelines: National Comprehensive Cancer Network.  Oncology (Huntingt) 1998;12:183-218.

Pollack R, Brennan M, Lawrence W, Jr: Society of Surgical Oncology practice guidelines: Soft-tissue sarcoma surgical practice guidelines.  Oncology (Huntingt) 1997;11:1327-1332.

 

61.      An 18-year-old boy has had pain in the right knee for the past 6 months.  Examination reveals some fullness behind the knee but no significant palpable soft-tissue mass.  There is no effusion, and he has full knee range of motion.  The remainder of the examination is unremarkable.  A radiograph and MRI scans are shown in Figures 33a through 33c, and biopsy specimens are shown in Figures 33d and 33e.  What is the most likely diagnosis?

 

1-         Parosteal osteosarcoma

2-         Classic osteogenic sarcoma

3-         Ewing’s sarcoma

4-         Osteochondroma

5-         Chondrosarcoma

 

PREFERRED RESPONSE: 1

 

DISCUSSION: The patient has parosteal osteosarcoma.  The posterior aspect of the distal femur is the typical location for this variant of osteogenic sarcoma.  The imaging studies indicate a surface lesion with no involvement of the adjacent intramedullary canal.  The histologic appearance is that of a low-grade fibroblastic osteosarcoma, consisting of relatively mature bone and a bland fibroblastic stroma lacking cytologic atypia and mitotic activity.  A cartilaginous component is also frequently seen.  Classic osteosarcoma typically has a more aggressive radiologic and histologic appearance.  Sessile osteochondromas, while common behind the knee, have a presence of hematopoietic marrow and fat.  The cartilage found in the associated cartilaginous cap is oriented.  Chondrosarcomas are more typical in an older age group and have a histologic pattern consisting of malignant chondroid. 

 

REFERENCES: Wold LA, et al: Atlas of Orthopaedic Pathology.  Philadelphia, PA, WB Saunders, 1990, pp 20-21.

Unni KK, Dahlin DC, Beabout JW, Ivins JC: Parosteal osteogenic sarcoma.  Cancer 1976;37:2466-2475.

 

62.     A 30-year-old patient has wrist pain.  A radiograph and biopsy specimen are shown in Figures 34a and 34b.  What is the most likely diagnosis?

 

1-         Aneurysmal bone cyst

2-         Giant cell tumor

3-         Brown tumor

4-         Pigmented villonodular synovitis

5-         Telangiectatic osteosarcoma

 

PREFERRED RESPONSE: 1

 

DISCUSSION: Aneurysmal bone cysts typically present as radiolucent lesions with an expansile remodeled cortex.  The histologic appearance consists of blood-filled lakes surrounded by a benign lining that contains fibroblasts, giant cells, and hemosiderin.  Although the other lesions are in the radiographic differential diagnosis, these histologic findings indicate an aneurysmal bone cyst.

 

REFERENCES: Bieselker JL, Marcove RC, Huvos AG, Mike V: Aneurysmal bone cyst: A Clinico-pathologic study of 66 cases.  Cancer 1973;26:615.

Martinez V, Sissons HA: A review of 123 cases including primary lesions and those secondary to other bone pathology.  Cancer 1988;61:2291.

 

63.     Mutations of what gene are associated with subsequent development of osteosarcoma?

 

1-         NF1 (neurofibromatosis)

2-         Rb (retinoblastoma)

3-         VEGF (vascular endothelial growth factor)

4-         E-Cadherin

5-         EWS-FLI1 (11,22 translocation)

 

PREFERRED RESPONSE: 2

 

DISCUSSION: The mutation of the retinoblastoma gene has been associated with an increased prevalence of osteosarcoma.  The mutation resulting in EWS-FLI1 is associated with Ewing’s sarcoma.  The other mutations are associated with tumor formation and proliferation but not necessarily with osteosarcoma formation.

 

REFERENCES: Menendez LR (ed): Orthopaedic Knowledge Update: Musculoskeletal Tumors.  Rosemont, IL, American Academy of Orthopaedic Surgeons, 2002, p 4. 

Scholz R, Kabisch H, Delling G, Winkler K: Homozygous deletion within the retinoblastoma gene in a native osteosarcoma specimen of a patient cured of a retinoblastoma of both eyes.  Pediatr Hematol Oncol 1990;72:65.

Hovig E, Lothe R, Farrants G, et al: Chromosome thirteen alterations in osteosarcoma cell lines derived from a patient with previous retinoblastoma.  Cancer Genet Cytogenet 1991;57:31-40.

 

 

64.     A 12-year-old girl has painless bowing of the tibia.  Radiographs and a biopsy specimen are shown in Figures 35a through 35c.  What is the most likely diagnosis?

 

1-         Osteofibrous dysplasia

2-         Adamantinoma

3-         Osteosarcoma

4-         Ewing’s sarcoma

5-         Fibrous dysplasia

 

PREFERRED RESPONSE: 1

 

DISCUSSION: The patient has osteofibrous dysplasia.  The radiographic differential diagnosis includes osteofibrous dysplasia, fibrous dysplasia, and adamantinoma.  Histology shows a fibro-osseous lesion with prominent osteoblastic rimming but a lack of epithelial nests.   Adamantinoma is a low-grade malignancy that typically is located in the anterior tibial cortex and has a soap bubble appearance.  Histologically, it is similar to osteofibrous dysplasia but includes epithelial nests of cells.  Treatment requires resection.  Fibrous dysplasia usually does not require biopsy; however, in this patient the radiographs do not distinguish it from adamantinoma.  The radiographic findings are not typical of Ewing’s sarcoma or osteosarcoma.  Repeat biopsy should be considered if clinical or radiographic features change.

 

REFERENCE: Mirra J: Bone Tumors: Clinical, Radiologic, and Pathologic Correlations.  Philadelphia, PA, Lea & Febiger, 1989, vol 2, ch 18.

65.     A 54-year-old man with metastatic renal cell carcinoma has had increasing pain in the left hip for the past 6 weeks.  A radiograph is shown in Figure 36.  Prophylactic stabilization will most likely result in

 

1-         a more technically difficult operation.

2-         increased intraoperative blood loss.

3-         increased overall survival rate.

4-         increased pain compared to an actual pathologic hip fracture.

5-         faster postoperative recovery and decreased hospitalization time.

 

PREFERRED RESPONSE: 5

 

DISCUSSION: Prophylactic stabilization of impending fractures does not directly affect the overall survival rate, but it does improve factors related to intraoperative and postoperative complications and decreased recovery time.

 

REFERENCES: Mirels H: Metastatic disease in long bones: A proposed scoring system for diagnosing impending pathologic fractures.  Clin Orthop 1989;249:256-264.

Harrington KD: Impending pathologic fractures from metastatic malignancy: Evaluation and management.  Instr Course Lect 1986;35:357-381.

 

 

66.     Which of the following is considered the treatment of choice for a 3-cm chondroblastoma of the distal femoral epiphysis with no intra-articular extension?

 

1-         Observation

2-         Curettage and bone grafting

3-         Wide local excision with radiation therapy

4-         Radiation therapy only

5-         Radiofrequency ablation

 

PREFERRED RESPONSE: 2

 

DISCUSSION: Curettage and bone grafting typically are the preferred treatment of chondroblastoma, yielding acceptable local recurrence rates of less than 10%.  Some surgeons advocate adjuvant therapies such as phenol, liquid nitrogen, or argon beam coagulation.  Untreated, these lesions can destroy bone and invade the joint to a significant degree.  Large intra-articular lesions may require major joint reconstruction.  Wide local excision is rarely required to control the tumor.  Radiation therapy is indicated only in unresectable lesions.

 

REFERENCES: Springfield DS, Capanna R, Gherlinzoni F, et al: Chondroblastoma: A review of seventy cases.  J Bone Joint Surg Am 1985;67:748.

Simon M, Springfield D, et al: Chrondroblastoma: Surgery for Bone and Soft Tissue Tumors.  Philadelphia, PA, Lippincott Raven, 1998, p 190.

 

67.      A radiograph, MRI scans, and a biopsy specimen of a 9-year-old boy with thigh pain are shown in Figures 37a through 37d.  Management should consist of

 

1-         external beam radiation.

2-         hemipelvectomy.

3-         hip disarticulation.

4-         wide resection of the femur and reconstruction.

5-         antibiotics.

 

PREFERRED RESPONSE: 4

 

DISCUSSION: The patient has Ewing’s sarcoma.  Management options for local tumor control include radiation therapy, resection, or a combination; however, in this patient wide resection is preferred over radiation therapy.  Radiation therapy is associated with damage to the growth plate, pathologic fracture, radiation-induced sarcomas, and a local recurrence rate of approximately 10% to 12%.  Radiation therapy is used for positive margins, unresectable tumors, or for tumors that have a poor response to chemotherapy.  Amputation is not necessary since the tumor is resectable.  Chemotherapy has improved overall survival rates to over 60% of patients.

 

REFERENCES: Sailer SL: The role of radiation therapy in localized Ewing’ sarcoma.  Semin Radiat Oncol 1997;7:225-235.

Shankar AG, Pinkerton CR, Atra A, Ashley S, Lewis I, Spooner D, et al: Local therapy and other factors influencing site of relapse in patients with localised Ewing’s sarcoma.  United Kingdom Children’s Cancer Study Group (UKCCSG).  Eur J Cancer 1999;35:1698-1704.

Carrie C, Mascard E, Gomez F, Habrand JL, Alapetite C, Oberlin O, et al: Nonmetastatic pelvic Ewing sarcoma: Report of the French society of pediatric oncology.  Med Pediatr Oncol 1999;33:444-449.

Terek RM, Brien EW, Marcove RC, Meyers PA, Lane JM, Healey JH: Treatment of femoral Ewing’s sarcoma.  Cancer 1996;78:70-78.

 

 

68.     A 47-year-old woman has had a 1-month history of left hip and medial thigh pain that is exacerbated by sitting.  Laboratory studies show a total protein level of 8.2 g/dL (normal 6.0 to 8.0) and an immunoglobulin G (IGG) level of 2,130 mg/dL (normal 562 to 1,835).  A radiograph, CT scan, and biopsy specimen are shown in Figures 38a through 38c.  What is the most likely diagnosis?

 

1-         Osteomyelitis

2-         Lymphoma

3-         Myeloma

4-         Ewing’s sarcoma

5-         Osteosarcoma

 

PREFERRED RESPONSE: 3

 

DISCUSSION: The laboratory studies and histology are both consistent with myeloma. Infection should show white blood cells other than plasma cells on histology.  Lymphoma would show lymphocytes, not plasma cells.  The lack of bone formation on the imaging studies and the lack of osteoid on histology rule out osteosarcoma.  The cells have too much cytoplasm and nuclear chromatin to be Ewing’s sarcoma cells.

 

REFERENCE: Mirra J: Bone Tumors: Clinical, Radiologic, and Pathologic Correlations.  Philadelphia, PA, Lea & Febiger, 1989, vol 2, ch 16.

 

69.     A 14-year-old boy has an anteromedial distal thigh mass.  A radiograph and MRI scan are shown in Figures 39a and 39b.  An open biopsy of the mass should include

 

1-         bone obtained through a knee arthrotomy with creation of a portal that will be used for retrograde femoral nailing.

2-         bone obtained by going through the mass.

3-         bone obtained through a posterior approach, dissecting out and protecting the sciatic nerve and popliteal artery.

4-         soft tissue obtained through a longitudinal incision centered over the mass.

5-         soft tissue obtained through a transverse incision on the medial thigh.

 

PREFERRED RESPONSE: 4

 

DISCUSSION: Biopsy of the soft-tissue component is often diagnostic.  Alternatively, in centers with pathologists familiar with bone tumors, needle biopsy is usually successful.  The principles of biopsy of bone tumors include  avoiding contamination of uninvolved structures and compartments, taking the most direct path to the tumors, making an excisable biopsy tract, and obtaining diagnostic tissue.  Transverse biopsy incisions should be avoided because they hinder the definitive surgical procedure.

 

REFERENCES: Peabody TD, Simon MA: Making the diagnosis: Keys to a successful biopsy in children with bone and soft-tissue tumors.  Orthop Clin North Am 1996;27:453-459.

Mankin HJ, Mankin CJ, Simon MA: The hazards of the biopsy, revisited.  Members of the Musculoskeletal Tumor Society.  J Bone Joint Surg Am 1996;78:656-663.

Skrzynski MC, Biermann JS, Montag A, Simon MA: Diagnostic accuracy and charge-savings of outpatient core needle biopsy compared with open biopsy of musculoskeletal tumors.  J Bone Joint Surg Am 1996;78:644-649.

 

 

70.     A 60-year-old man has pain at the tip of the index finger.  A radiograph and biopsy specimen are shown in Figures 40a and 40b.  Management should consist of

 

1-         surgical debridement and antibiotics.

2-         curettage and bone graft.

3-         amputation through the distal interphalangeal joint.

4-         amputation of the second ray.

5-         radiation therapy.

 

PREFERRED RESPONSE: 3

 

DISCUSSION: The radiograph and histology findings are most consistent with squamous cell carcinoma.  This tumor is best treated with wide surgical resection margins alone in the absence of metastasis; in this patient, management should consist of amputation through the distal interphalangeal joint.  The other treatments are not indicated.

 

REFERENCE: Soltani K, Krunic A: Non melanoma skin neoplasms, in Vokes E, Golomb H (eds): Oncologic Therapies, ed 2.  Berlin, Germany, Springer, pp 646-647.

 

 

71.      An infant is born with a mass that involves both the volar and dorsal compartments of the left arm.  A clinical photograph and biopsy specimen are shown in Figures 41a and 41b.  What is the best initial course of action?

 

1-         Debulking

2-         Wide resection with vascular and nerve grafting

3-         Above-elbow amputation

4-         Chemotherapy

5-         Radiation therapy

 

PREFERRED RESPONSE: 4

 

DISCUSSION: The patient has infantile fibrosarcoma.  For unresectable lesions, the treatment of choice is chemotherapy with vincristine, actinomycin-D, and cyclophosphamide, followed by excision if there is an adequate decrease in the size of the lesion.

 

REFERENCE: Kurkchubasche AG, Halvorson EG, Forman EN, Terek RM, Ferguson WS: The role of preoperative chemotherapy in the treatment of infantile fibrosarcoma.  J Pediatr Surgery 2000;35:880-883.

 

72.     Which of the following processes does not account for decreased hematopoiesis in patients with metastatic disease?

 

1-         External beam radiation

2-         A decrease in cellular calcium

3-         Cytotoxic systemic chemotherapy

4-         Decreased iron use secondary to anemia of chronic disease

5-         Marrow replacement by tumor cells

 

PREFERRED RESPONSE: 2

 

DISCUSSION: Paucytopenia is a common problem in patients with metastatic disease.  Causes include chemotherapy, external beam radiation, marrow replacement by tumor, and anemia of chronic disease.  There is no correlation with decreased calcium and a decrease in hematopoiesis.  Supportive care with granulocyte-colony stimulating factor (G-CSF) and neupogen can stimulate hematopoiesis.

 

REFERENCE: Frassica FJ, Gitelis S, Sim FH: Metastic bone disease: General principles, pathophysiology, evaluation, and biopsy.  Instr Course Lect 1992;41:293-300.

 

73.      A 43-year-old woman has an enlarging mass in the left groin.  A radiograph, CT scan, and a biopsy specimen are shown in Figures 42a through 42c.  Treatment should consist of

 

1-         wide resection of the ischiopelvic region and acetabulum with reconstruction.

2-         wide resection of the ischiopelvic region inferior to the acetabulum.

3-         observation.

4-         external beam radiation.

5-         classic hemipelvectomy.

 

PREFERRED RESPONSE: 2

 

DISCUSSION: The patient has a pelvic chondrosarcoma.  The radiograph shows a lytic bone lesion emanating from the left inferior pubic ramus and extending into the soft tissues.  Punctate calcifications are revealed on the radiograph and CT scan.  The histology is consistent with a malignant cartilage lesion.  Appropriate treatment for a pelvic chondrosarcoma is wide resection.  In this location, wide resection of the ischiopelvic region (type 3 internal hemipelvectomy) is the treatment of choice.  A type 2 internal hemipelvectomy involves resection of the periacetabular region.  A type 1 internal hemipelvectomy involves resection of the ilium.  No reconstruction is required for a type 3 resection.  A classic hemipelvectomy is not necessary because the tumor can be removed with an adequate margin while maintaining the neurovascular structures and hip joint. 

 

REFERENCES: Pring M, Weber KL, Unni K, Sim FH: Chondrosarcoma of the pelvis: A review of sixty-four cases.  J Bone Joint Surg Am 2001;83:1630-1642.

Sheth DS, Yasko AW, Johnson ME, Ayala AG, Murray JA, Romsdahl MM: Chondrosarcoma of the pelvis: Prognostic factors for 67 patients treated with definitive surgery.  Cancer 1996;78:745-750.

Enneking WF, Dunham WK: Resection and reconstruction for primary neoplasms involving the innominate bone.  J Bone Joint Surg Am 1978;60:731-746.

 

74.      A 66-year-old man has a high-grade angiosarcoma of the right tibia.  A radiograph is shown in Figure 43.  Treatment should consist of

 

1-         amputation.

2-         wide resection.

3-         radical resection.

4-         radiation therapy.

5-         chemotherapy.

 

PREFERRED RESPONSE: 1

 

DISCUSSION: Angiosarcoma is a locally aggressive sarcoma.  The radiograph shows extensive multiple discontinuous lesions throughout the entire tibia.  The extent of bone involvement precludes resection; therefore, the treatment of choice is amputation, either above the knee or through the knee.  Radiation therapy is not needed after amputation, and chemotherapy remains investigational for soft-tissue sarcoma.

 

REFERENCE: Simon MA, Springfield DA: Surgery for Bone and Soft Tissue Tumors.  Philadelphia, PA, Lippincott-Raven, 1998, ch 29.

 

75.      Figures 44a and 44b show the radiographs of a 28-year-old woman who has had progressive hip pain for the past 3 months.  What is the most likely diagnosis?

 

1-         Enchondroma

2-         Osteochondroma

3-         Chondrosarcoma

4-         Ollier’s disease

5-         Maffucci’s syndrome

 

PREFERRED RESPONSE: 3

 

DISCUSSION: The patient has multiple hereditary exostoses and a secondary chondrosarcoma arising from a proximal femoral exostosis.  The radiograph of the knee shows multiple osteochondromas typical in a patient with multiple hereditary exostoses.  Patients with this diagnosis are at an increased risk for malignant degeneration of an osteochondroma.  The lateral radiograph of the hip shows a bony lesion emanating from the anterior aspect of the femoral neck that is not well defined in the surrounding soft tissues.  There are punctate calcifications and a large soft-tissue mass.  The most likely diagnosis is a secondary chondrosarcoma developing from a benign osteochondroma.  An enchondroma is an intramedullary benign cartilage lesion.  Ollier’s disease and Maffucci’s syndrome involve multiple enchondromas. 

 

REFERENCES: Scarborough M, Moreau G: Benign cartilage tumors.  Orthop Clin North Am 1996;27:583-589.

Garrison R, Unni K, McLeod RA, Pritchard DJ, Dahlin DC: Chondrosarcoma arising in osteochondroma.  Cancer 1982;49:1890-1897.

 

76.      A 19-year-old girl has had pain and swelling in the right ankle for the past 4 months.  She denies any history of trauma.  Examination reveals a small soft-tissue mass over the anterior aspect of the ankle and slight pain with range of motion of the ankle joint.  The examination is otherwise unremarkable.  A radiograph and MRI scan are shown in Figures 45a and 45b, and biopsy specimens are shown in Figures 45c and 45d.  What is the most likely diagnosis?

 

1-         Osteogenic sarcoma

2-         Ewing’s sarcoma

3-         Giant cell tumor of bone

4-         Aneurysmal bone cyst

5-         Metastatic adenocarcinoma

 

PREFERRED RESPONSE: 3

 

DISCUSSION: Giant cell tumors typically occur in a juxta-articular location involving the epiphysis and metaphysis of long bones, usually eccentric in the bone.  The radiographs show a destructive process within the distal tibia and an associated soft-tissue mass.  The histology shows multinucleated giant cells in a bland matrix with a few scattered mitoses.  Osteosarcoma can have a similar destructive appearance but a very different histologic pattern with osteoid production.  Ewing’s sarcoma also can have a diffuse destructive process in the bone.  The histologic pattern of Ewing’s sarcoma is diffuse round blue cells.  Aneurysmal bone cysts typically are seen as a fluid-filled lesion on imaging studies and have only a scant amount of giant cells histologically.  Metastatic adenocarcinoma does not demonstrate the pattern shown in the patient’s histology specimen.

 

REFERENCES: Wold LA, et al: Atlas of Orthopaedic Pathology.  Philadelphia, PA, WB Saunders, 1990, pp 198-199.

Simon M, et al: Surgery for Bone and Soft Tissue Tumors.  Philadelphia, PA, Lippincott Raven, 1998, pp 200-202.

 

77.      A 13-year-old boy has had a painless mass in the arm for the past 2 months.  An MRI scan and biopsy specimens are shown in Figures 46a through 46c.  What is the most likely diagnosis?

 

1-         Desmoid tumor

2-         Rhabdomyosarcoma

3-         Nodular fasciitis

4-         Malignant fibrous histiocytoma

5-         Lipoma

 

PREFERRED RESPONSE: 3

 

DISCUSSION: Nodular fasciitis is a benign soft-tissue lesion that usually arises from the fascia and is often misdiagnosed as a sarcoma.  Desmoid tumors (aggressive fibromatosis) are also benign tumors with a greater tendency for local recurrence.  Desmoid tumors have more spindle-shaped fibroblasts in an abundant collagenous matrix.  Malignant fibrous histiocytoma is a hypercellular pleomorphic sarcoma more commonly found in adults.  The histology is not consistant with a fatty tumor.

 

REFERENCE: Bernstein KE, Lattes R: Nodular (pseudosarcomatous) fasciitis, a nonrecurrent lesion: Clinicopathologic study of 134 cases.  Cancer 1982;49:1668-1678.

 

78.      A 20-year-old man has a symptomatic lesion of fibrous dysplasia in the femoral neck.  Management should consist of

 

1-         curettage, cortical strutting, and internal fixation.

2-         curettage only.

3-         observation.

4-         neoadjuvant chemotherapy.

5-         radiation therapy.

 

PREFERRED RESPONSE: 1

 

DISCUSSION: Fibrous dysplasia in the femoral neck frequently warrants treatment because of the risk of pathologic fracture.  Cortical strut grafts reduce the risk of local recurrence compared with cancellous bone grafting.  Because of the consequences associated with fracture in this location, prophylactic fixation is recommended.  Radiation therapy and chemotherapy are not used for this benign condition.

 

REFERENCES: Simon M, et al: Surgery for Bone and Soft Tissue Tumors.  Philadelphia, PA, Lippincott Raven, 1998, pp 197.

Enneking WF, Gearen PF: Fibrous dysplasia of the femoral neck: Treatment by cortical bone grafting.  J Bone Joint Surg Am 1986;68:1415.

 

79.      In addition to radiographs of the primary lesion and chest, MRI of the primary lesion, and CT of the chest, staging studies for Ewing’s sarcoma should include which of the following?

 

1-         Bone scan and gallium scan

2-         Bone scan and sentinel node biopsy

3-         Bone scan and bone marrow biopsy

4-         Bone marrow and sentinel node biopsy

5-         Gallium scan and sentinel node biopsy

 

 

PREFERRED RESPONSE: 3

 

DISCUSSION: A bone scan and bone marrow biopsy are part of the staging studies for Ewing’s sarcoma.  Whole body MRI and PET scans are investigational and show promise of greater sensitivity than a bone scan. 

 

REFERENCES: Schleiermacher G, Peter M, Oberlin O, Philip T, Rubie H, Mechinaud F, et al: Increased risk of systemic relapses associated with bone marrow micrometastasis and circulating tumor cells in localized ewing tumor.  J Clin Oncol 2003;21:85-91.

Daldrup-Link HE, Franzius C, Link TM, Laukamp D, Sciuk J, Jurgens H, et al: Whole-body MR imaging for detection of bone metastases in children and young adults: Comparison with skeletal scintigraphy and FDG PET.  Am J Roentgenol 2001;177:229-236.

 

 

80.     Which of the following conditions is transmitted by an autosomal dominant trait?

 

1-         Multiple hereditary exostosis

2-         Ollier’s disease

3-         Li-Fraumeni syndrome

4-         Maffucci’s syndrome

5-         Retinoblastoma

 

PREFERRED RESPONSE: 1

 

DISCUSSION: Multiple hereditary exostosis is transmitted by an autosomal dominant trait.  Li-Fraumeni syndrome and retinoblastoma are autosomal recessive or associated with autosomal recessive mutations.  No genetic predisposition to Ollier’s disease or Maffucci’s syndrome has been identified. 

 

REFERENCES: Mirra J (ed): Bone Tumors: Clinical, Radiologic and Pathologic Correlations.  Philadelphia, PA, Lea and Febiger, 1989, p 1627.

Menendez LR (ed): Orthopaedic Knowledge Update: Musculoskeletal Tumors.  Rosemont, IL, American Academy of Orthopaedic Surgeons, 2002, p 107.

 

81.      A previously healthy 13-year-old girl has had thigh pain for the past 3 weeks.  The radiograph shown in Figure 47a reveals a lesion in the right femur.  A bone scan and CT scan of the chest show no evidence of other lesions.  A biopsy specimen is shown in Figure 47b.  What is the most likely diagnosis?

 

1-         Ewing’s sarcoma

2-         Lymphoma of bone

3-         Metastatic neuroblastoma

4-         Langerhans cell histiocytosis

5-         Osteomyelitis

 

PREFERRED RESPONSE: 4

 

DISCUSSION: The patient has Langerhans cell histiocytosis that may be solitary (eosinophilic granuloma) or associated with systemic illness (Hand-Schuller-Christian disease and Letterer-Siwe disease).  The solitary form of the disease, eosinophilic granuloma, typically affects patients in the first three decades of life.  Radiographically, it is characterized as a well-defined, lytic, “punched out” intramedullary lesion.  Histologically, two cell types, eosinophils and Langerhans cells, are seen.  The Langerhans cells are seen as mononuclear histiocyte-like cells with oval nuclei with well-defined round or oval cytoplasm.  A prominent nuclear groove can be seen in most of the nuclei (coffee bean nuclei).  A mixture of inflammatory cells and lipid-laden foam cells with nuclear debris may be present as well.  The lack of nuclear atypia and atypical mitoses excludes malignant conditions such as Ewing’s sarcoma, lymphoma of bone, and metastatic neuroblastoma.  The lack of acute inflammatory cells excludes the diagnosis of osteomyelitis.  The eosinophils have bi-lobed nuclei and granular eosinophilic cytoplasm.

 

REFERENCES: Dorfman H, Czerniak B: Bone Tumors.  St Louis, MO, Mosby, 1988.

Mirra, JM: Bone Tumors: Clinical, Radiologic, and Pathologic Correlations.  Philadelphia, PA, Lea & Febiger, 1989.

 

82.     A 50-year-old man with metastatic renal cell carcinoma has right hip pain.  A radiograph and CT scan are shown in Figures 48a and 48b.  The first step in management should consist of

 

1-         administration of intravenous bisphosphonates.

2-         cemented right total hip arthroplasty with acetabular reconstruction.

3-         external beam radiation.

4-         embolization of the lesion.

5-         percutaneous cementoplasty.

 

PREFERRED RESPONSE: 4

 

DISCUSSION: These lesions are extremely vascular and can cause uncontrolled intraoperative bleeding; therefore embolization is the appropriate first treatment.  Because the radiograph and CT scan show a lytic lesion in the supra-acetabular region that affects the weight-bearing dome and medial wall, the next step in treatment would most likely be a total hip arthroplasty and acetabular reconstruction.  Treatment with bisphosphonates and radiation therapy will not prevent an acetabular fracture.  Cementoplasty is an emerging technique in which cement is injected percutaneously into a lesion, but no long-term results have been reported.  Radiofrequency ablation of bone metastases is also an emerging technique that provides palliative pain control.

 

REFERENCES: Layalle I, Flandroy P, Trotteur G, Dondelinger RF: Arterial embolization of bone metastases: Is it worthwhile?  J Belge Radiol 1998;81:223-225.

Chatziioannou AN, Johnson ME, Pneumaticos SG, Lawrence DD, Carrasco CH: Preoperative embolization of bone metastases from renal cell carcinoma.  Eur Radiol 2000;10:593-596.

 

83.     What is the most common benign bone tumor in childhood?

 

1-         Unicameral bone cyst

2-         Fibrous dysplasia

3-         Nonossifying fibroma

4-         Aneurysmal bone cyst

5-         Chondromyxoid fibroma

 

PREFERRED RESPONSE: 3

 

DISCUSSION: The most common benign bone tumor in childhood is a nonossifying fibroma.  It is estimated that 30% of children have a nonossifying fibroma.  In most patients, the lesion is not identified until a radiograph is obtained for unrelated reasons.  Similarly, most identified cases of fibrous cortical defect are not biopsied because the radiographic and clinical presentations are diagnostic.

 

REFERENCES: Aboulafia AJ, Kennon RE, Jelinek JS: Benign bone tumors of childhood.  J Am Acad Orthop Surg 1999;7:377-388.

Biermann JS: Common benign lesions of bone in children and adolescents.  J Pediatr Orthop 2002;22:268-273.

 

84.     A 13-year-old patient has foot drop and lateral knee pain.  AP and lateral radiographs and an MRI scan are shown in Figures 49a through 49c.  A biopsy specimen is shown in Figure 49d.  What is the preferred method of treatment?

 

1-         Wide resection alone

2-         Chemotherapy and radiation therapy

3-         Chemotherapy and wide resection

4-         Above-knee amputation

5-         Through-knee amputation

 

PREFERRED RESPONSE: 3

 

DISCUSSION: The “sunburst” radiographic appearance suggests an osteosarcoma, and the histologic findings confirm the diagnosis with malignant cells surrounded by pink osteoid.  MRI scans are not particularly helpful in the diagnosis of osteosarcoma but are mandatory for surgical planning.  Osteosarcomas are high-grade sarcomas that are best treated with chemotherapy and wide resection.  Even though the peroneal nerve is involved, limb salvage is indicated.  Survival after limb salvage is equivalent to amputation, with better function.

 

REFERENCES: Goorin AM, Abelson HT, Frei E: Osteosarcoma: Fifteen years later.  N Engl J Med 1985;313:1637.

Link MP, Goorin AM, Miser AW, et al: The effect of adjuvant chemotherapy on relapse-free survival in patients with osteosarcoma of the extremity.  N Engl J Med 1986;314:1600.

Davis AM, Bell RS, Goodwin PJ: Prognostic factors in osteosarcoma: A critical review.  J Clin Oncol 1994;12:423.

 

85.     A 13-year-old girl is diagnosed with a stage IIB osteosarcoma of the proximal tibia.  Following neoadjuvant chemotherapy, local control should consist of

 

1-         wide resection and reconstruction.

2-         wide resection and radiation therapy.

3-         radiation therapy.

4-         curettage and bone grafting.

5-         observation.

 

PREFERRED RESPONSE: 1

 

DISCUSSION: Local control of osteosarcoma consists of wide resection and reconstruction.  Radiation therapy is not recommended except in unresectable lesions or for palliation.  Curettage and bone grafting result in intralesional resection with an unacceptable high rate of local recurrence.  Chemotherapy alone is not adequate for local control.

 

REFERENCES: Simon M, Springfield D, et al: Osteogenic Sarcoma: Surgery for Bone and Soft Tissue.  Philadelphia, PA, Lippincott Raven, 1998, p 274. 

Wold LA, et al: Osteogenic Sarcoma: Atlas of Orthopaedic Pathology.  Philadelphia, PA, WB Saunders, 1990, pp 14-15.

 

86.     A 23-year-old woman has had vague left knee pain for the past 6 months.  A radiograph and CT scan are shown in Figures 50a and 50b.  What is the most likely diagnosis?

 

1-         Myositis ossificans

2-         Osteochondroma

3-         Parosteal osteosarcoma

4-         Dedifferentiated chondrosarcoma

5-         Tumoral calcinosis

 

PREFERRED RESPONSE: 3

 

DISCUSSION: The radiographic appearance of the lesion emanating from the posterior cortex of the left distal femur is consistent with a surface bone-producing lesion; therefore, the most likely diagnosis is a parosteal osteosarcoma.  In an osteochondroma, the cortex and medullary cavity of the lesion are in continuity with that of the native bone.  A dedifferentiated chondrosarcoma has histologic components of a high-grade sarcoma plus a benign or low-grade malignant cartilage tumor.  Tumoral calcinosis is characterized by amorphous calcium in the soft tissues and does not emanate from the bone itself.  While often confused with parosteal osteosarcoma, myositis ossificans is usually more mature at the periphery of the lesion rather than the center.  In addition, myositis ossificans does not involve the underlying cortex but remains separate from the bone.

 

REFERENCES: Unni KK: Dahlin’s Bone Tumors: General Aspects and Data on 11,087 Cases, ed 5.  Philadelphia, PA, Lippincott-Raven, 1996, pp 185-196.

Okada K, Frassica FJ, Sim FH, Beabout JW, Bond JR, Unni KK: Parosteal osteosarcoma.  A clinicopathological study.  J Bone Joint Surg Am 1994;76:366-378.

 

87.      What is the preferred treatment of a patient with breast cancer and a pathologic fracture of the clavicle in her dominant arm?

 

1-         Surgical stabilization

2-         Sling

3-         Physical therapy

4-         Wide resection

5-         Radiofrequency ablation

 

PREFERRED RESPONSE: 2

 

DISCUSSION: Closed management should be attempted for upper extremity pathologic fractures, particularly the clavicle.  If nonunion or pain persists,  surgery may be indicated.  Radiofrequency ablation is not indicated for subcutaneous bones.  Early motion is likely to cause increased pain and disability.

 

REFERENCES: Weber KC, Lewis VO, Randall RL, Lee AK, Springfield D: An approach to the management of the patient with metastatic bone disease.  Instr Course Lect 2004;53:663-676.

Menendez LR (ed): Orthopaedic Knowledge Update: Musculoskeletal Tumors.  Rosemont, IL, American Academy of Orthopaedic Surgeons 2002, p 331.

 

88.     A 14-year-old boy has an asymptomatic mass on the right arm.  MRI scans and biopsy specimens are shown in Figures 51a through 51d.  Immunostaining is positive for desmin.  Additional staging studies should include

 

1-         a bone scan and thallium scan.

2-         a regional lymph node and bone marrow biopsy.

3-         MRI of the brain.

4-         intravenous pyelogram and renal ultrasound.

5-         parathyroid hormone and serum calcium levels.

 

PREFERRED RESPONSE: 2

 

DISCUSSION: The patient has rhabdomyosarcoma.  Axilliary node and bone marrow biopsy are part of the staging because about 12% of patients with rhabdomyosarcoma of the extremity have evidence of lymph nodes metastases at presentation.  Bone marrow metastases have been shown to portend a worse prognosis.

 

REFERENCES: Lawrence W, Jr., Hays DM, Heyn R, Tefft M, Crist W, Beltangady M, et al: Lymphatic metastases with childhood rhabdomyosarcoma: A report from the Intergroup Rhabdomyosarcoma Study.  Cancer 1987;60:910-915.

Schleiermacher G, Peter M, Oberlin O, et al: Increased risk of systemic relapses associated with bone marrow micrometastasis and circulating tumor cells in localized ewing tumor.   J Clin Oncol 2003;21:85-91.

 

89.     A 15-year-old girl has left knee pain and an enlarging mass in the distal thigh.  AP and lateral radiographs are shown in Figures 52a and 52b, and a biopsy specimen is shown in Figure 52c.  What is the most likely diagnosis?

 

1-         Osteosarcoma

2-         Ewing’s sarcoma

3-         Chondrosarcoma

4-         Metastatic neuroblastoma

5-         Osteochondroma

 

PREFERRED RESPONSE: 1

 

DISCUSSION: A bone-producing lesion in the metaphysis of an adolescent is most likely an osteosarcoma. The radiographs show a distal femoral bone-producing lesion extending into the surrounding soft tissues.  The histologic appearance consists of pleomorphic cells producing osteoid.    Ewing’s sarcoma and metastatic neuroblastoma do not produce a matrix.  Chondrosarcoma is a radiographically destructive lesion with calcification and cartilage cells on histologic section.  An osteochondroma is a benign cartilage lesion that is continuous with the medullary cavity of the underlying bone and extends into a bony lesion and covered by a cartilage cap.

 

REFERENCES: Unni KK: Dahlin’s Bone Tumors: General Aspects and Data on 11,087 Cases, ed 5.  Philadelphia, PA, Lippincott-Raven, 1996, pp 143-160.

Gibbs CP, Weber K, Scarborough MT: Malignant bone tumors.  Instr Course Lect 2002;51:413-428.

 

90.     Survival rates for children with soft-tissue sarcoma other than rhabdomyosarcoma are best correlated with

 

1-         size of the tumor.

2-         histologic grade.

3-         histologic subtype.

4-         use of adjuvant chemotherapy.

5-         age of the patient.

 

PREFERRED RESPONSE: 2

 

DISCUSSION: In review of 154 patients with nonrhabdomyosarcoma, Rao reported that histologic grade, tumor invasiveness, and adequate surgical margin were the most important prognostic factors.  Histologic subtype, use of adjuvant chemotherapy, and patient age were not as important.  Size related to degree of invasiveness was not statistically significant.

 

REFERENCES: Rao BN: Nonrhabdomyosarcoma in children: Prognostic factors influencing survival.  Semin Surg Oncol 1993;9:524-531.

Andrassy R, et al: Non-rhabdomyosarcoma Soft-Tissue Sarcomas: Pediatric Surgical Oncology.  Philadelphia, PA, WB Saunders, p 221.

 

91.      A 25-year-old woman has had pain and stiffness in her knee following a motor vehicle accident 9 months ago.  The radiograph, CT scan, MRI scan, and biopsy specimen are shown in Figures 53a through 53d.  What is the most likely diagnosis?

 

1-         Osteochondroma

2-         Osteoblastoma

3-         Osteomyelitis

4-         Heterotopic ossification

5-         Parosteal osteosarcoma

 

PREFERRED RESPONSE: 4

 

DISCUSSION: Heterotopic ossification may occur spontaneously or following trauma.  The imaging studies and histology reveal mature fatty bone marrow and trabecular bone.  Osteochondromas are cortically based with the medullary canal extending into the lesion.  This is not evident in this patient.  Also, no obvious cartilage cap is present.  Parosteal osteosarcoma commonly occurs in the posterior distal femoral cortex but is ruled out by the lack of the typical fibrous stromal cells forming the low-grade malignant osteoid.  The histology and clinical presentation eliminate osteomyelitis and osteoblastoma. 

 

REFERENCES: Horne LT, Blue BA: Intra-articular heterotopic ossification in the knee following intramedullary nailing of the fractured femur using a retrograde method.  J Orthop Trauma 1999;13:385-388.

Stannard JP, Wilson TC, Sheils TM, McGwin G Jr, Volgas DA, Alonso JE: Heterotopic ossification associated with knee dislocation.  Arthroscopy 2002;18:835-839.

Mills WJ, Tejwani N: Heterotopic ossification after knee dislocation: The predictive value of the injury severity score.  J Orthop Trauma 2003;17:338-345.

 

92.     Which of the following lesions most closely resembles Ewing’s sarcoma histologically?

 

1-         Metastatic neuroblastoma

2-         Osteosarcoma

3-         Osteomyelitis

4-         Fibrous dysplasia

5-         Fracture healing

 

PREFERRED RESPONSE: 1

 

DISCUSSION: Ewing’s sarcoma is characterized by small round blue cells.  Lesions with a similar appearance include lymphoma, primitive neuroectodermal tumor, rhabdomyosarcoma, small cell lung tumor, and metastatic neuroblastoma.  Karyotyping, immunohistochemistry, and electron microscopy can help differentiate these lesions.

 

REFERENCE: Wold LE, McLeod RA, Sim FH, Unni KK: Atlas of Orthop Pathology.  Philadelphia, PA, WB Saunders, 1990.

 

93.     Primary chondrosarcoma of bone most commonly occurs in which of the following locations?

 

1-         Hand

2-         Knee

3-         Shoulder

4-         Spine

5-         Pelvis

 

PREFERRED RESPONSE: 5

 

DISCUSSION: The most common location of chondrosarcoma is the pelvis (30%), followed by the proximal femur (20%) and shoulder girdle (15%).  Chondrosarcoma rarely affects the spine or hand.

 

REFERENCES: Lee FY, Mankin HJ, Fondren G, et al: Chondrosarcoma of bone: An assessment of outcome.  J Bone Joint Surg Am 1999;81:326-338.

Simon M, Springfield D, et al: Chondrosarcoma: Surgery for Bone and Soft Tissue Tumors.  Philadelphia, PA, Lippincott Raven, 1998, p 276.

 

94.     An 18-year-old man has had an enlarging mass in his hand for the past 3 months.  Radiographs, an MRI scan, and biopsy specimens are shown in Figures 54a through 54d.  What is the most likely diagnosis?

 

1-         Bizarre parosteal osteochondromatous proliferation (BPOP)

2-         Osteosarcoma

3-         Ewing’s sarcoma

4-         Enchondroma

5-         Infection

 

PREFERRED RESPONSE: 1

 

DISCUSSION: Also known as Nora’s lesion, BPOP is a benign osteocartilaginous tumor that almost always occurs in the hands and feet; one occurrence each in the femur and tibia has been reported.  Although local recurrence is common after excision, metastases have not been reported.

 

REFERENCES: Abramovici L, Steiner GC: Bizarre parosteal osteochondromatous proliferation (Nora’s lesion): A retrospective study of 12 cases, 2 arising in long bones.  Hum Pathol 2002;33:1205-1210.

Nora FE, Dahlin DC, Beabout JW: Bizarre parosteal osteochondromatous proliferations of the hands and feet.  Am J Surg Pathol 1983;7:245-250.

 

95.     A 55-year-old man has had a mass in his right thigh for the past 2 months.  An MRI scan and biopsy specimens are shown in Figures 55a through 55c.  What is the most likely diagnosis?

 

1-         Extraskeletal myxoid chondrosarcoma

2-         Rhabdomyosarcoma

3-         Malignant fibrous histiocytoma

4-         Myxoma

5-         Liposarcoma

 

PREFERRED RESPONSE: 1

 

DISCUSSION: The histology shows extraskeletal myxoid chondrosarcoma, characterized by abundant blue myxoid matrix with cords and nests of small tumor cells.  Treatment consists of wide resection.  Despite the name, hyaline cartilage is not a common component of these tumors.  Adult rhabdomyosarcoma and malignant fibrous histiocytoma are highly pleomorphic sarcomas often containing multinucleated giant cells.  Myxoid liposarcoma contains a prominent capillary network and lipoblasts.  Myxoma is less cellular than extraskeletal myxoid chondrosarcoma and does not have a cord-like arrangement of tumor cells. 

 

REFERENCE: Kawaguchi S, Wada T, Nagoya S, Ikeda T, Isu K, Yamashiro K, et al: Extraskeletal myxoid chondrosarcoma.  Cancer 2003;97:1285-1292.

 

96.     Figures 56a through 56c show the radiograph, CT scan, and biopsy specimen of a 44-year-old man who underwent chemotherapy and radiation therapy for lymphoma of the distal femur 20 years ago.  His current problem is most likely related to

 

1-         steroid-induced osteonecrosis.

2-         radiation therapy with secondary malignancy.

3-         recurrence of the lymphoma.

4-         radiation osteitis.

5-         a primary lung tumor.

 

PREFERRED RESPONSE: 2

 

DISCUSSION: The patient has changes consistent with radiation therapy to the femur, including osteopenia and an aggressive appearing neoplasm.  The tumor is most likely a radiation-induced sarcoma.  This is more likely than recurrent lymphoma at this late date.  It is not related to steroid use or a primary lung tumor.

 

REFERENCES: Mirra J (ed): Bone Tumors: Clinical, Radiologic and Pathologic Correlations.  Philadelphia, PA, Lea and Febiger, 1989, p 353.

Huvos A, Woodard H, Cahan W, et al: Postradiation osteogenic sarcoma of bone and soft tissue.  A clinical pathologic study of 66 Patients.  Cancer 1985;55:1244.

 

97.      What is the most common reason an individual with a malignant soft-tissue tumor in the extremities seeks medical attention?

 

1-         Incidental finding on imaging tests

2-         Pain

3-         Presence of a mass

4-         Deformity

5-         Neurologic symptoms

 

PREFERRED RESPONSE: 3

 

DISCUSSION: Unlike malignant bone tumors, malignant soft-tissue tumors usually are asymptomatic and present with the presence of a mass.  Malignant soft-tissue tumors enlarge by centrifugal growth, creating a mass while compressing surrounding tissue.  Symptoms may develop as the result of direct compression on neurovascular structures as the tumor enlarges.  This is especially true in the pelvis where the tumor can enlarge appreciably without being noticed.  However, in the extremities, the tumor is most often apparent before neurologic symptoms develop.  An asymptomatic mass is not necessarily benign; therefore, biopsy should not be delayed.  It is uncommon for a malignant soft-tissue mass to be discovered incidentally.  Soft-tissue tumors are not typically apparent on radiographs; they are best identified with MRI. 

 

REFERENCES: Brouns F, Stas M, De Wever I: Delay in diagnosis of soft tissue sarcomas.  Eur J Surg Oncol 2003;29:440-445.

Rougraff B: The diagnosis and management of soft tissue sarcomas of the extremities in the adult.  Curr Probl Cancer 1999;23:1-50.

Sim FH, Frassica FJ, Frassica DA: Soft-tissue tumors: Diagnosis, evaluation, and management.  J Am Acad Orthop Surg 1994;2:202-211.

 

98.     Exostoses in which of the following anatomic locations is the most likely to undergo malignant transformation in a patient with multiple hereditary exostosis (MHE)?

 

1-         Distal humerus

2-         Distal radius

3-         Distal femur

4-         Lumbar spine

5-         Scapula

 

PREFERRED RESPONSE: 5

 

DISCUSSION: Although osteochondromas can occur in almost every bone in patients with MHE, proximally located lesions are more likely to undergo malignant transformation.  Annual radiographs of the shoulder girdles and pelvis are indicated in patients with MHE.  Any enlarging osteochondromas are a concern as possible malignancies.

 

REFERENCES: Peterson HA: Multiple hereditary osteochondromata.  Clin Orthop 1989;239:222.

McCornack EB: The surgical management of hereditary multiple exostosis.  Orthop Rev 1981;10:57.

 

99.     Initial management of a pathologic fracture of the humerus secondary to a unicameral bone cyst should include

 

1-         open curettage and bone grafting.

2-         intralesional steroid injection.

3-         percutaneous grafting with a bone graft substitute.

4-         immobilization until the fracture heals.

5-         MRI.

 

PREFERRED RESPONSE: 4

 

DISCUSSION: Most pathologic humeral fractures secondary to a unicameral bone cyst are minimally displaced and should be immobilized and allowed to heal.  Persistent and/or progressive lesions may require treatment.  Various treatments of unicameral bone cysts have been described.  Acceptable treatment options include curettage and bone grafting, intralesional steroid injection, and percutaneous grafting with bone graft substitutes.  MRI is not indicated when the diagnosis of unicameral bone cyst is known.

 

REFERENCES: Wilkins RM: Unicameral bone cysts.  J Am Acad Orthop Surg 2000;8:217-224.

Bensahel H, Jehanno P, Desgrippes Y, Pennecot GF: Solitary bone cyst: Controversies and treatment.  J Pediatr Orthop B 1998;7:257-261.

 

100.   An 11-year-old child has Ewing’s sarcoma of the femoral diaphysis with a small soft-tissue mass.  Staging studies show no evidence of metastases.  Treatment should

consist of

 

1-         chemotherapy and radiation therapy to the femur.

2-         chemotherapy and wide resection.

3-         chemotherapy and above-knee amputation.

4-         hip disarticulation.

5-         wide resection.

 

PREFERRED RESPONSE: 2

 

DISCUSSION: The use of chemotherapy has dramatically improved survival rates of patients with Ewing’s sarcoma.  Local disease is best handled with wide resection to decrease local recurrence and to avoid the complications of radiation therapy (ie, secondary sarcomas).  Radiation therapy alone is reserved for unresectable lesions or poor surgical margins.  Amputation generally is not necessary.

 

REFERENCES: Toni A, Neff JR, Sudanese A, et al: The role of surgical therapy in patients with non-metastatic Ewing’s sarcoma of the limbs.  Clin Orthop 1991;286:225.

Picci P, Rougraff BT, Bacci G, et al: Prognostic significance of histopathologic response to chemotherapy in non-metastatic Ewing’s sarcoma of the extremities.  J Clin Oncol 1993;11:1763.

Gibbs CP Jr, Weber K, Scarborough MT: Malignant Bone Tumors.  Instr Course Lect 2002;51:413-428.

 

 

FOR ALL MCQS CLICK THE LINK ORTHO MCQ BANK